Você está na página 1de 53

NCLEX Questions 1 intubation or a tracheostomy in the event of

Answers and Rationale further or complete obstruction.


1. Answer: A: 45-year-old African American 10. Answer: C: Bedwetting
attorney In children, fatigue and bed wetting are the chief
The incidence of hypertension is greater among complaints that prompt parents to take their child
African Americans than other groups in the US. for evaluation. Bed wetting in a school age child
The incidence among the Hispanic population is is readily detected by the parents.
rising. 11. Answer: B: Chlamydia
2. Answer: A: Gastric lavage PRN Chlamydial infections are one of the most
Removing as much of the drug as possible is the frequent causes of salpingitis or pelvic
first step in treatment for this drug overdose. inflammatory disease.
This is best done by gastric lavage. The next 12. Answer: C: An adolescent who has been on
drug to give would be activated charcoal, then pain medications for terminal cancer with an
mucomyst and lastly the IV fluids. initial assessment finding of pinpoint pupils and
3. Answer: B: thrombus formation a relaxed respiratory rate of 10
Thrombus formation in the coronary arteries is a Nurses who are floated to other units should be
potential problem in the initial 24 hours after a assigned to a client who has minimal anticipated
cardiac catheterization. A falling BP occurs immediate complications of their problem. The
along with hemorrhage of the insertion site client in option C exhibits opioid toxicity with the
which is associated with the first 12 hours after pinpoint pupils and has the least risk of
the procedure. complications to occur in the near future.
4. Answer: C: Manage pain 13. Answer: C: Avoiding very heavy meals
The immediate goal of therapy is to alleviate the Eating large, heavy meals can pull blood away
clients pain. from the heart for digestion and is dangerous for
5. Answer: D: Yearly weight gain of about 5.5 the client with coronary artery disease.
pounds per year 14. Answer: C: The level of drug is 100 mL at 8
School age children gain about 5.5 pounds each AM and is 80 mL at noon
year and increase about 2 inches in height. The minimal dose of 10 mL per hour which
6. Answer: A: go get a blood pressure check would be 40 mL given in a four (4) hour period.
within the next 48 to 72 hours Only 60 mL should be left at noon. The pump is
The blood pressure reading is moderately high not functioning when more than expected
with the need to have it rechecked in a few days. medicine is left in the container.
The client states it is usually much lower. Thus 15. Answer: B: Spinal column manipulation
a concern exists for complications such as The theory underlying chiropractic is that
stroke. However, immediate check by the interference with transmission of mental
provider of care is not warranted. Waiting 2 impulses between the brain and body organs
months or a week for follow-up is too long. produces diseases. Such interference is caused
7. Answer: A: A middle-aged client with a history by misalignment of the vertebrae. Manipulation
of being ventilator dependent for over 7 years reduces the subluxation.
and admitted with bacterial pneumonia five days 16. Answer: A: Decrease in level of
ago consciousness
The best candidate for discharge is one who has A further decrease in the level of consciousness
had a chronic condition and is most familiar with would be indicative of a further progression of
their care. This client in option A is most likely the CVA.
stable and could continue medication therapy at 17. Answer: C: Moist, productive cough Option c
home. is a later sign.
8. Answer: A: Should be taken in the morning Noisy respirations and a dry non-productive
Thyroid supplement should be taken in the cough are commonly the first of the respiratory
morning to minimize the side effects of insomnia signs to appear in a newly diagnosed client with
9. Answer: D: Notify the health care provider of cystic fibrosis (CF). The other options are the
the childs status earliest findings. CF is an inherited (genetic)
These findings suggest a medical emergency condition affecting the cells that produce mucus,
and may be due to epiglottitis. Any child with an sweat, saliva and digestive juices. Normally,
acute onset of an inflammatory response in the these secretions are thin and slippery, but in CF,
mouth and throat should receive immediate a defective gene causes the secretions to
attention in a facility equipped to perform become thick and sticky. Instead of acting as a
lubricant, the secretions plug up tubes, ducts, forces of yin and yang.
and passageways, especially in the pancreas 27. Answer: C: Force fluids and reassess blood
and lungs. Respiratory failure is the most pressure
dangerous consequence of CF. Postural hypotension, a decrease in systolic
18. Answer: B: Send him to the emergency room blood pressure of more than 15 mmHg and an
for evaluation increase in heart rate of more than 15 percent
This client requires immediate evaluation. A usually accompanied by dizziness indicate
delay in treatment could result in further volume depletion, inadequate vasoconstrictor
deterioration and harm. Home care nurses must mechanisms, and autonomic insufficiency.
prioritize interventions based on assessment 28. Answer: D. Left ventricular functioning
findings that are in the clients best interest. The catheter is placed in the pulmonary artery.
19. Answer: D: No special orders are necessary Information regarding left ventricular function is
for this examination obtained when the catheter balloon is inflated.
No special preparation is necessary for this 29. Answer: B. Initiate high-quality chest
examination. compressions
20. Answer: B: When you can climb 2 flights of As per new guidelines, the American Heart
stairs without problems, it is generally safe. Association recommends beginning CPR with
There is a risk of cardiac rupture at the point of chest compression (rather than checking for the
the myocardial infarction for about six (6) weeks. airway first). Start CPR with 30 chest
Scar tissue should form about that time. Waiting compressions before checking the airway and
until the client can tolerate climbing stairs is the giving rescue breaths. Starting with chest
usual advice given by health care providers. compressions first applies to adults, children,
21. Answer: B: A teenager who got signed beard and infants needing CPR, but not
while camping newborns. CPR can keep oxygenated blood
This client is in the greatest danger with a flowing to the brain and other vital organs until
potential of respiratory distress. Any client with more definitive medical treatment can restore a
singed facial hair has been exposed to heat or normal heart rhythm.
fire in close range that could have caused 30. Answer: A: Blood pressure 94/60
serious damage to the interior of the lungs. Note Both medications decrease the heart rate.
that the interior linings of the lungs have no Metoprolol affects blood pressure. Therefore,
nerve fibers so the client will not be aware of the heart rate and blood pressure must be within
swelling. normal range (HR 60-100; systolic BP over 100)
22. Answer: C: I understand the need to use in order to safely administer both medications.
those new skills. 31. Answer: C: Inspiratory grunt
Erikson describes the stage of the toddler as Inspiratory grunting is abnormal and may be a
being the time when there is normally an sign of respiratory distress in this infant.
increase in autonomy. The child needs to use 32. Answer: D: Progressive placental
motor skills to explore the environment. insufficiency
23. Answer: A: Verify correct placement of the The placenta functions less efficiently as the
tube pregnancy continues beyond 42 weeks.
Proper placement of the tube prevents Immediate and long-term effects may be related
aspiration. to hypoxia.
24. Answer: C: Tall peaked T waves 33. Answer: B: I just cant catch my breath
A tall peaked T wave is a sign of hyperkalemia. over the past few minutes and I think I am in
The health care provider should be notified grave danger.
regarding discontinuing the medication. The nurse would be concerned about all of
25. Answer: A: All striated muscles these comments. However, the most life
Rhabdomyosarcoma is the most common threatening is option B. Clients who have had
childrens soft tissue sarcoma. It originates in hip or knee surgery are at greatest risk for
striated (skeletal) muscles and can be found development of postoperative pulmonary
anywhere in the body. The clue is in the middle embolism. Sudden dyspnea and tachycardia are
of the word and is myo which typically means classic findings of pulmonary embolism. Muscle
muscle. spasms do not require immediate attention.
26. Answer: D: Restore yin and yang Option C may indicate a urinary tract infection.
For followers of Chinese medicine, health is And option D requires further investigation and
maintained through the balance between the is not life threatening.
34. Answer: D: Decreased appetite a high risk of causing permanent
Lasix causes a loss of potassium if a damage to the fetus.
supplement is not taken. Signs and symptoms of C: Celecoxib (Celebrex). Large doses
hypokalemia include anorexia, fatigue, nausea, cause birth defects in rabbits; not known
decreased GI motility, muscle weakness, if the effect on people is the same.
dysrhythmias. D: Clonidine (Catapres). Crosses the
35. Answer: C: Gravida 3 para 1 placenta but no adverse fetal effects
Gravida is the number of pregnancies and Parity have been observed.
is the number of pregnancies that reach viability E: Transdermal nicotine (Habitrol).
(not the number of fetuses). Thus, for this Nicotine replacement products have
woman, she is now pregnant, had 2 prior been assigned to pregnancy category C
pregnancies, and 1 viable birth (twins). (nicotine gum) and category D
36. Answer: B: Improve the (transdermal patches, inhalers, and
clients nutritional status spray nicotine products).
The goal of clinical management in a client with F: Clofazimine (Lamprene). Clofazimine
venous stasis ulcers is to promote healing. This has been assigned to pregnancy
only can be accomplished with proper nutrition. category C.
The other answers are correct, but without 2. Answers: A, B, C, D, and E.
proper nutrition, the other interventions would be Photosensitivity is an extreme sensitivity to
of little help. ultraviolet (UV) rays from the sun and other light
37. Answer: D: Have the client empty bladder sources. A type of photosensitivity called
The first step in the process is to have the client Phototoxic reactions are caused when
void prior to administering the pre-operative medications in the body interact with UV rays
medication. The other actions follow this initial from the sun. Antiinfectives are the most
step in this sequence: D, C, A and then B. common cause of this type of reaction.
38. Answer: A: Specific feedback is given as 3. Answer: D. Aspirin
close to the event as possible D. Aspirin is not known to cause
Feedback is most useful when given discoloration of the urine.
immediately. Positive behavior is strengthened A: Sulfasalazine may discolor the urine
through immediate feedback, and it is easier to or skin to an orange-yellow color.
modify problem behaviors if the standards are B: Levodopa may discolor the urine,
clearly understood. saliva, or sweat to a dark brown color.
39. Answer: B: Lead to dehydration C: Phenolphthalein can discolor the
The client must take in adequate fluids before urine to a red color.
and during exercise periods. 4. Answer: A. Corgard
40. Answer: C: We have safety bars installed in A: Nadolol (Corgard): Stored at room
the bathroom and have 24-hour alarms on the temperature between 59 to 86 F (15
doors. and 30 C) away from heat, moisture,
Ensuring safety of the client with increasing and light. Do not store in the bathroom
memory loss is a priority of home care. Note all and keep bottle tightly closed.
options are correct statements. However, safety B: Humulin N injection: If unopened (not
is most important to reinforce. in use) store Humulin N in the fridge and
use until expiration date; or store at
NCLEX Questions 2 room temperature and use within 31
1. Answers: A, and B. days. If opened (in-use), store the vial in
A: Warfarin (Coumadin). Has a a refrigerator or at room temperature
pregnancy category X and associated and use within 31 days. Store the
with central nervous system defects, injection pen at room temperature (do
spontaneous abortion, stillbirth, not refrigerate) and use within 14 days.
prematurity, hemorrhage, and ocular Keep it in its original container protected
defects when given anytime during from heat and light. Do not draw insulin
pregnancy and a fetal warfarin from a vial into a syringe until you are
syndrome when given during the first ready to give an injection. Do not freeze
trimester. insulin or store it near the cooling
B: Finasteride (Propecia, Proscar). Also element in a refrigerator. Throw away
has a pregnancy category X which has any insulin that has been frozen.
C: Urokinase (Kinlytic): Refrigerate at 2 hypertension, and a high risk of
8C. cardiovascular morbidity and mortality.
D: Epoetin alfa IV (Epogen): Vials D: Somatic neuropathy affects the whole
should be stored at 2C to 8C (36F to body and presents with diverse clinical
46F); Do not freeze. Do not shake. pictures, most common is the
Protect from light. development of diabetic foot followed by
5. Answer: D. IgG diabetic ulceration and possible
D: IgG is the only immunoglobulin that amputation.
can cross the placental barrier. 8. Answer: B. Anorexia nervosa
A: IgA antibodies protect body surfaces B: All of the clinical signs and symptoms
that are exposed to outside foreign point to a condition of anorexia nervosa.
substances. The key feature of anorexia nervosa is
B: IgD antibodies are found in small self-imposed starvation, resulting from a
amounts in the tissues that line the belly distorted body image and an intense,
or chest. irrational fear of gaining weight, even
C: IgE antibodies cause the body to when the patient is emaciated. Anorexia
react against foreign substances such nervosa may include refusal to eat
as pollen, spores, animal dander. accompanied by compulsive exercising,
6. Answer: B. Start prophylactic AZT treatment self-induced vomiting, or laxative or
B: Azidothymidine (AZT) treatment is diuretic abuse.
the most critical intervention. It is an C: On the other hand, bulimia
antiretroviral medication used to prevent nervosa features binge eating followed
and treat HIV/AIDS by reducing the by a feeling of guilt, humiliation, and
replication of the virus. self-deprecation. These feelings cause
C: Pentamidine is an antimicrobial the patient to engage in self-induced
medication given to prevent and treat vomiting, use of laxatives or diuretics.
pneumocystis pneumonia. A: Multiple sclerosis (MS) is a
A and D: Other interventions mentioned demyelinating disease in which the
are to be done later. insulating covers of the nerve cells in
7. Answer: C. Autonomic neuropathy the brain and spinal cord are damaged.
C: Autonomic neuropathy (also known D: Systemic sclerosis or systemic
as Diabetic Autonomic Neuropathy) scleroderma is an autoimmune disease
affects the autonomic nerves, which of the connective tissue.
control the bladder, intestinal tract, and 9. Answer: B. Hypercalcemia
genitals, among other organs. Paralysis B: Hypercalcaemia can cause polyuria,
of the bladder is a common symptom of severe abdominal pain, and confusion.
this type of neuropathy. A: Diverticulosis is a condition that
A: Atherosclerosis, or hardening of the develops when pouches (diverticula)
arteries, is a condition in which plaque form in the wall of the large intestine;
builds up inside the arteries. Plaque is most people dont have symptoms.
made of cholesterol, fatty substances, C: Hypocalcemia is low calcium levels in
cellular waste products, calcium and the blood; it is asymptomatic in mild
fibrin (a clotting material in the blood). forms but can cause paresthesia,
B: Diabetic nephropathy (DN) is typically tetany, muscle cramps, and carpopedal
defined by macroalbuminuriathat is, a spasms in severe hypocalcemia.
urinary albumin excretion of more than D: Irritable bowel syndrome is a
300 mg in a 24-hour collectionor widespread condition involving recurrent
macroalbuminuria and abnormal renal abdominal pain and diarrhea or
function as represented by an constipation, often associated with
abnormality in serum creatinine, stress, depression, anxiety, or previous
calculated creatinine clearance, or intestinal infection.
glomerular filtration rate (GFR). 10. Answer: C. RH negative, RH positive
Clinically, diabetic nephropathy is Rhogam prevents the production of anti-RH
characterized by a progressive increase antibodies in the mother that has a Rh positive
in proteinuria and decline in GFR, fetus.
11. Answer: D. The effects of PKU are The skin would be dry and not oily.
reversible. 17. Answer: D. Observe for signs of bleeding.
Phenylketonuria (PKU) is an inherited disorder D: Bleeding is the priority concern for a
that increases the levels of phenylalanine (a client taking thrombolytic medication.
building block of proteins) in the blood. If PKU is A and B: Are monitored but are not the
not treated, phenylalanine can build up to primary concern.
harmful levels in the body, causing intellectual C: is not related to the use of
disability and other serious health problems. The medication.
signs and symptoms of PKU vary from mild to 18. Answer: A. Green vegetables and liver
severe. The most severe form of this disorder is Green vegetables and liver are a great source of
known as classic PKU. Infants with classic PKU folic acid.
appear normal until they are a few months old. 19. Answer: D. Cl. difficile
Without treatment, these children develop Cl. difficile has not been linked to meningitis.
a permanent intellectual disability. Seizures, 20. Answer: D. The life span of RBC is 120
delayed development, behavioral problems, and days.
psychiatric disorders are also common. Red blood cells have a lifespan of 120 in the
Untreated individuals may have a musty or body.
mouse-like odor as a side effect of excess 21. Answer: B. Upon admit
phenylalanine in the body. Children with classic Discharge education begins upon admit.
PKU tend to have lighter skin and hair than 22. Answer: B. Initiative vs. guilt
unaffected family members and are also likely to Initiative vs. guilt- 3-6 years old
have skin disorders such as eczema. The 23. Answer: C. Autonomy vs. shame
effects of PKU stay with the infant throughout Autonomy vs Shame and doubt is at 12-18
their life (via Genetic Home Reference). months old
12. Answer: A. Onset of pulmonary edema 24. Answer: D. Intimacy vs. isolation
Aspirin overdose can lead to metabolic Intimacy vs. isolation- 18-35 years old
acidosis and cause pulmonary edema 25. Answer: B. 13-year-old female: 105 BPM,
development. 22 RPM, 105/50 mmHg
Early symptoms of aspirin poisoning HR and Respirations are slightly increased. BP
also include tinnitus, hyperventilation, is down.
vomiting, dehydration, and fever. Late 26. Answer: A. Elavil
signs include drowsiness, bizarre A: Amitriptyline (Elavil) is a tricyclic
behavior, unsteady walking, and coma. antidepressant and used to treat
Abnormal breathing caused by aspirin symptoms of depression.
poisoning is usually rapid and deep. B: Calcitonin is used to treat
Pulmonary edema may be related to an osteoporosis in women who have been
increase in permeability within the in menopause.
capillaries of the lung leading to protein C: Pergolide mesylate (Permax) is used
leakage and transudation of fluid in in the treatment of Parkinsons disease.
both renal and pulmonary tissues. The D: Verapamil (Calan) is a calcium
alteration in renal tubule permeability channel blocker.
may lead to a change in colloid osmotic 27. Answer: D. Multiple Sclerosis
pressure and thus facilitate pulmonary Erythromycin is used to treat conditions A-C.
edema (via Medscape). 28. Answer: D. Migraines
13. Answer: D. Provide a secure environment for Answer choices A-C were symptoms of acute
the patient. hyperkalemia.
This patients safety is your primary concern. 29. Answer: C. Weight gain
14. Answer: C. Cough following bronchodilator Weight loss would be expected.
utilization 30. Answer: A. Increased appetite
The bronchodilator will allow a more productive Loss of appetite would be expected.
cough. 31. Answer: D. Haemophilus aegyptius
15. Answer: B. Weight gain A: is linked to Plague
Weight gain due to fluid accumulation is B: is linked to peptic ulcers
associated with heart failure and congenital C: is linked to Cholera.
heart defects. 32. Answer: A. Borrelia burgdorferi
16. Answer: C. Oily skin B: is linked to Rheumatic fever
C: is linked to Anthrax Hyperglycemia and ketoacidosis do not cause
D: is linked to Endocarditis. confusion and shakiness.
33. Answer: D. CT scan 4. Answer: A. Bowel perforation
A CT scan would be performed for further Bowel perforation is the most serious
investigation of the hemiparesis. complication of fiberoptic colonoscopy.
34. Answer: C. Thyroid function tests Important signs include progressive abdominal
Weight gain and poor temperature tolerance pain, fever, chills, and tachycardia, which
indicate something may be wrong with the indicate advancing peritonitis. Viral
thyroid function. gastroenteritis and colon cancer do not cause
35. Answer: C. Blood cultures these symptoms. Diverticulitis may cause pain,
Blood cultures would be performed to fever, and chills, but is far less serious than
investigate the fever and rash symptoms. perforation and peritonitis.
36. Answer: A. Blood sugar check 5. Answers: A, B, and C.
With a history of diabetes, the first response Prothrombin time, partial thromboplastin time,
should be to check blood sugar levels. and platelet count are all included in coagulation
37. Answer: C. The overall mental and physical studies. The hemoglobin level, though important
abilities of the child. information prior to an invasive procedure like
Age is not the greatest factor in potty training. liver biopsy, does not assess coagulation.
The overall mental and physical abilities of the 6. Answer: B. Contaminated food.
child are the most important factor. Hepatitis A is the only type that is transmitted by
38. Answer: C. Contact the Poison Control the fecal-oral route through contaminated food.
Center quickly Hepatitis B, C, and D are transmitted through
The poison control center will have an exact infected bodily fluids.
plan of action for this child. 7. Answer: A. A history of hepatitis C five
39. Answer: C. Vastus lateralis years previously.
Vastus lateralis is the most appropriate location. Hepatitis C is a viral infection transmitted
40. Answer: D. Ask the father who is in the room through bodily fluids, such as blood, causing
the childs name. inflammation of the liver. Patients with hepatitis
In this case, you can determine the name of the C may not donate blood for transfusion due to
child by the fathers statement. You should not the high risk of infection in the recipient.
withhold the medication from the child after Cholecystitis (gallbladder disease),
identification. diverticulosis, and history of Crohns disease do
not preclude blood donation.
NCLEX Questions 3 8. Answer: A. Naproxen sodium (Naprosyn).
1. Answer: A. Elevated serum calcium. Naproxen sodium is a nonsteroidal anti-
The parathyroid glands regulate the calcium inflammatory drug that can cause inflammation
level in the blood. In hyperparathyroidism, the of the upper GI tract. For this reason, it is
serum calcium level will be elevated. Parathyroid contraindicated in a patient with gastritis.
hormone levels may be high or normal but not Calcium carbonate is used as an antacid for the
low. The body will lower the level of vitamin D in relief of indigestion and is not contraindicated.
an attempt to lower calcium. Urine calcium may Clarithromycin is an antibacterial often used for
be elevated, with calcium spilling over from the treatment of Helicobacter pylori in gastritis.
elevated serum levels. This may cause renal Furosemide is a loop diuretic and is NOT
stones. contraindicated in a patient with gastritis.
2. Answer: D. A restricted sodium diet. 9. Answer: D. The patient should limit fatty
A patient with Addisons disease requires normal foods.
dietary sodium to prevent excess fluid loss. Cholecystitis, inflammation of the gallbladder, is
Adequate caloric intake is recommended with a most commonly caused by the presence of
diet high in protein and complex carbohydrates, gallstones, which may block bile (necessary for
including grains. fat absorption) from entering the intestines.
3. Answer: C. Hypoglycemia. Patients should decrease dietary fat by limiting
A post-operative diabetic patient who is unable foods like fatty meats, fried foods, and creamy
to eat is likely to be suffering from hypoglycemia. desserts to avoid irritation of the gallbladder.
Confusion and shakiness are common 10. Answer: D. Air hunger.
symptoms. An anesthesia reaction would not Patients with pulmonary edema experience air
occur on the second post-operative day. hunger, anxiety, and agitation. Respiration is
fast and shallow and heart rate increases. infection if white blood cells are decreased.
Stridor is noisy breathing caused by laryngeal Transfusion of red cells is indicated for severe
swelling or spasm and is not associated with anemia.
pulmonary edema. 16. Answer: B. Repeated vomiting.
11. Answer: C. A patient with a history of Increased pressure caused by bleeding or
ventricular tachycardia and syncopal swelling within the skull can damage delicate
episodes. brain tissue and may become life threatening.
An automatic internal cardioverter-defibrillator Repeated vomiting can be an early sign of
delivers an electric shock to the heart to pressure as the vomit center within the medulla
terminate episodes of ventricular tachycardia is stimulated. The anterior fontanel is closed in a
and ventricular fibrillation. This is necessary in a 4-year-old child. Evidence of sleepiness at 10
patient with significant ventricular symptoms, PM is normal for a four-year-old. The average 4-
such as tachycardia resulting in syncope. A year-old child cannot read yet, so this too is
patient with myocardial infarction that resolved normal.
with no permanent cardiac damage would not be 17. Answer: A. Small blue-white spots are
a candidate. A patient recovering well from visible on the oral mucosa.
coronary bypass would not need the device. Kopliks spots are small blue-white spots visible
Atrial tachycardia is less serious and is treated on the oral mucosa and are characteristic of
conservatively with medication and measles infection. The body rash typically
cardioversion as a last resort. begins on the face and travels downward. High
12. Answer: B. The patient has a pacemaker. fever is often present. Tear drop on a rose
The implanted pacemaker will interfere with the petal refers to the lesions found in varicella
magnetic fields of the MRI scanner and may be (chicken pox).
deactivated by them. Shellfish/iodine allergy is 18. Answer: C. Petechiae occur on the soft
not a contraindication because the contrast used palate.
in MRI scanning is not iodine-based. Open MRI Petechiae on the soft palate are characteristic of
scanners and anti-anxiety medications are rubella infection. Choices A, B, and D are
available for patients with claustrophobia. characteristic of scarlet fever, a result of group A
Psychiatric medication is not a contraindication Streptococcus infection.
to MRI scanning. 19. Answer: B. The dose is too low.
13. Answer: B. The patient suddenly This child weighs 30 kg, and the pediatric dose
complains of chest pain and shortness of of diphenhydramine is 5 mg/kg/day (5 X 30 =
breath. 150/day). Therefore, the correct dose is 150
Typical symptoms of pulmonary embolism mg/day. Divided into 3 doses per day, the child
include chest pain, shortness of breath, and should receive 50 mg 3 times a day rather than
severe anxiety. The physician should be notified 25 mg 3 times a day. Dosage should not be
immediately. A patient with pulmonary embolism titrated based on symptoms without consulting a
will not be sleepy or have a cough with crackles physician.
on exam. A patient with fever, chills and loss of 20. Answer: D. Normally, the testes descend
appetite may be developing pneumonia. by one year of age.
14. Answer: C. The patient will be admitted to Normally, the testes descend by one year of
the surgical unit and resection will be age. In young infants, it is common for the testes
scheduled. to retract into the inguinal canal when the
A rapidly enlarging abdominal aortic aneurysm is environment is cold or the cremasteric reflex is
at significant risk of rupture and should be stimulated. Exam should be done in a warm
resected as soon as possible. No other room with warm hands. It is most likely that both
appropriate treatment options currently exist. testes are present and will descend by a year. If
15. Answer: D. Check for signs of bleeding, not, a full assessment will determine the
including examination of urine and stool for appropriate treatment.
blood. 21. Answer: C. The tumor extended beyond
A platelet count of 25,000/microliter is severely the kidney but was completely resected.
thrombocytopenic and should prompt the The staging of Wilms tumor is confirmed at
initiation of bleeding precautions, including surgery as follows: Stage I, the tumor is limited
monitoring urine and stool for evidence of to the kidney and completely resected; stage II,
bleeding. Monitoring for fever and requiring the tumor extends beyond the kidney but is
protective clothing are indicated to prevent completely resected; stage III, residual
nonhematogenous tumor is confined to the oxygen demand in the leg muscles exceeds the
abdomen; stage IV, hematogenous metastasis oxygen supply. This most often occurs during
has occurred with spread beyond the abdomen; activity when demand increases in muscle
and stage V, bilateral renal involvement is tissue. The tissue becomes hypoxic, causing
present at diagnosis. cramping, weakness, and discomfort.
22. Answers: A, B, and C 28. Answer: C. Avoid crossing the legs.
Acute glomerulonephritis is characterized by Patients with peripheral vascular disease should
high urine specific gravity related to oliguria as avoid crossing the legs because this can impede
well as dark tea-colored urine caused by large blood flow. Walking barefoot is not advised, as
amounts of red blood cells. There is periorbital foot protection is important to avoid trauma that
edema, but generalized edema is seen in may lead to serious infection. Heating pads can
nephrotic syndrome, not acute cause injury, which can also increase the risk of
glomerulonephritis. infection. Skin lesions at risk for infection should
23. Answer: B. Prior infection with group A be examined and treated by a physician.
Streptococcus within the past 10-14 days. 29. Answer: C. A young woman.
Acute glomerulonephritis is most commonly Raynauds disease is most common in young
caused by the immune response to a prior upper women and is frequently associated with
respiratory infection with group A Streptococcus. rheumatologic disorders, such as lupus and
Glomerular inflammation occurs about 10-14 rheumatoid arthritis.
days after the infection, resulting in scant, dark 30. Answer: B. Pulmonary embolism due to
urine and retention of body fluid. Periorbital deep vein thrombosis (DVT).
edema and hypertension are common signs at In a hospitalized patient on prolonged bed rest,
diagnosis. the most likely cause of sudden onset shortness
24. Answer: C. No treatment is necessary; of breath and chest pain is pulmonary embolism.
the fluid is reabsorbing normally. Pregnancy and prolonged inactivity both
A hydrocele is a collection of fluid in the scrotum increase the risk of clot formation in the deep
that results from a patent tunica vaginalis. veins of the legs. These clots can then break
Illumination of the scrotum with a pocket light loose and travel to the lungs. Myocardial
demonstrates the clear fluid. In most cases the infarction and atherosclerosis are unlikely in a
fluid reabsorbs within the first few months of life 27-year-old woman, as is congestive heart
and no treatment is necessary. Massaging the failure due to fluid overload. There is no reason
area or placing the infant in a supine position to suspect an anxiety disorder in this patient.
would have no effect. Surgery is not indicated. Though anxiety is a possible cause of her
25. Answer: A. Inadequate tissue perfusion symptoms, the seriousness of pulmonary
leading to nerve damage. embolism demands that it be considered first.
Patients with peripheral vascular disease often 31. Answer: B. Cerebral hemorrhage.
sustain nerve damage as a result of inadequate Cerebral hemorrhage is a significant risk when
tissue perfusion. Fluid overload is not treating a stroke victim with thrombolytic therapy
characteristic of PVD. There is nothing to intended to dissolve a suspected clot. Success
indicate psychiatric disturbance in the patient. of the treatment demands that it be instituted as
Skin changes in PVD are secondary to soon as possible, often before the cause of
decreased tissue perfusion rather than primary stroke has been determined. Air embolus is not
inflammation. a concern. Thrombolytic therapy does not lead
26. Answer: A. Family history of heart to expansion of the clot, but to resolution, which
disease. is the intended effect.
Family history of heart disease is an inherited 32. Answer: A. Torticollis, with shortening of
risk factor that is not subject to lifestyle change. the sternocleidomastoid muscle.
Having a first-degree relative with heart disease In torticollis, the sternocleidomastoid muscle is
has been shown to significantly increase risk. contracted, limiting the range of motion of the
Overweight and smoking are risk factors that are neck and causing the chin to point to the
subject to lifestyle change and can reduce risk opposing side. In craniosynostosis one of the
significantly. Advancing age increases the risk of cranial sutures, often the sagittal, closes
atherosclerosis but is not a hereditary factor. prematurely, causing the head to grow in an
27. Answers: A, C, and D. abnormal shape. Plagiocephaly refers to the
Claudication describes the pain experienced by flattening of one side of the head, caused by the
a patient with peripheral vascular disease when infant being placed supine in the same position
over time. Hydrocephalus is caused by a build- The childs refusal to walk, combined with
up of cerebrospinal fluid in the brain resulting in swelling of the limb is suspicious for fracture.
large head size. Toddlers will often continue to walk on a muscle
33. Answer: C. The student experiences pain that is bruised or strained. The radius is found in
in the inferior aspect of the knee. the lower arm and is not relevant to this
Osgood-Schlatter disease occurs in adolescents question. Toddlers rarely feign injury to be
in rapid growth phase when the infrapatellar carried, and swelling indicates a physical injury.
ligament of the quadriceps muscle pulls on the 39. Answers: A, B, and D.
tibial tubercle, causing pain and swelling in the Delayed developmental milestones are
inferior aspect of the knee. Osgood-Schlatter characteristic of cerebral palsy, so regular
disease is commonly caused by activities that screening and intervention is essential. Because
require repeated use of the quadriceps, of injury to upper motor neurons, children may
including track and soccer. Swimming is not a have ocular and speech difficulties. Parent
likely cause. The condition is usually self-limited, support groups help families to share and cope.
responding to ice, rest, and analgesics. Physical therapy and other interventions can
Continued participation will worsen the condition minimize the extent of the delay in
and the symptoms. developmental milestones.
34. Answer: D. Scoliosis. 40. Answer: A. Duchennes is an X-linked
A check for scoliosis, a lateral deviation of the recessive disorder, so daughters have a 50%
spine, is an important part of the routine chance of being carriers and sons a 50%
adolescent exam. It is assessed by having the chance of developing the disease.
teen bend at the waist with arms dangling, while The recessive Duchenne gene is located on one
observing for lateral curvature and uneven rib of the two X chromosomes of a female carrier. If
level. Scoliosis is more common in female her son receives the X bearing the gene he will
adolescents. Choices A, B, and C are not part of be affected. Thus, there is a 50% chance of a
the routine adolescent exam. son being affected. Daughters are not affected,
35. Answer: C. Self-blame for the injury to but 50% are carriers because they inherit one
the child. copy of the defective gene from the mother. The
The profile of a parent at risk of abusive other X chromosome comes from the father,
behavior includes a tendency to blame the child who cannot be a carrier.
or others for the injury sustained. These parents
also have a high incidence of low self-esteem, NCLEX Questions 4
unemployment, unstable financial situation, and Answers and Rationale
single status. 1. Answer: A. Lessen the amount of cellular
36. Answer: C. Nonsteroidal anti- damage
inflammatory drugs are the first choice in Rapid continuous rewarming of a frostbite
treatment. primarily lessens cellular damage. It does not
Nonsteroidal anti-inflammatory drugs are prevent formation of blisters. It does promote
important first line treatment for juvenile movement, but this is not the primary reason for
idiopathic arthritis (formerly known as juvenile rapid rewarming. It might increase pain for a
rheumatoid arthritis). NSAIDs require 3-4 weeks short period of time as the feeling comes back
for the therapeutic anti-inflammatory effects to into the extremity; therefore, answers B, C, and
be realized. Half of children with the disorder D are incorrect.
recover without joint deformity and about a third 2. Answer: D. Filtering waste through a
will continue with symptoms into adulthood. dialyzing membrane
Physical activity is an integral part of therapy. Hemodialysis works by using a dialyzing
37. Answer: B. A blood culture is drawn. membrane to filter waste that has accumulated
Antibiotics must be started after the blood in the blood. It does not pass water through a
culture is drawn, as they may interfere with the dialyzing membrane nor does it eliminate
identification of the causative organism. The plasma proteins or lower the pH, so answers A,
blood count will reveal the presence of infection B, and C are incorrect.
but does not help identify an organism or guide 3. Answer: B. Contact the physician for an
antibiotic treatment. Parental presence is order for immune globulin
important for the adjustment of the child but not The client who is immunosuppressed and is
for the administration of medication. exposed to measles should be treated with
38. Answer: A. Possible fracture of the tibia. medications to boost his immunity to the virus.
An antibiotic or antiviral will not protect the client The client who is having a central venous
and it is too late to place the client in isolation, catheter removed should be told to hold his
so answers A, C, and D are incorrect. breath and bear down. This prevents air from
4. Answer: D. Infection requires skin-to-skin entering the line. Answers B, C, and D will not
contact and is prevented by hand washing, facilitate removal.
gloves, and a gown. 10. Answer: B. A history of streptococcal
The client with MRSA should be placed in infections
isolation. Gloves, a gown, and a mask should be Clients with a history of streptococcal infections
used when caring for the client and hand could have antibodies that render the
washing is very important. The door should streptokinase ineffective. There is no reason to
remain closed, but a negative-pressure room is assess the client for allergies to pineapples or
not necessary, so answers A and B are bananas, there is no correlation to the use of
incorrect. MRSA is spread by contact with blood phenytoin and streptokinase, and a history of
or body fluid or by touching the skin of the client. alcohol abuse is also not a factor in the order for
It is cultured from the nasal passages of the streptokinase; therefore, answers A, C, and D
client, so the client should be instructed to cover are incorrect.
his nose and mouth when he sneezes or 11. Answer: B. Flossing between the teeth
coughs. It is not necessary for the client to wear The client who is immune-suppressed and has
the mask at all times; the nurse should wear the bone marrow suppression should be taught not
mask, so answer C is incorrect. to floss his teeth because platelets are
5. Answer: B. The pain is due to peripheral decreased. Using oils and cream-based soaps is
nervous system interruptions. I will get you allowed, as is eating salt and using an
some pain medication. electric razor; therefore, answers A, C, and D
Pain related to phantom limb syndrome is due to are incorrect.
peripheral nervous system interruption. Answer: 12. Answer: A. Apply the new tie before
A. is incorrect because phantom limb pain can removing the old one.
last several months or indefinitely. Answer: C. is The best method and safest way to change the
incorrect because it is not psychological. It is ties of a tracheotomy is to apply the new ones
also not due to infections, as stated in answer D. before removing the old ones. Having a helper is
6. Answer: A. Head of the pancreas good, but the helper might not prevent the client
During a Whipple procedure the head of the from coughing out the tracheotomy. Answer: C.
pancreas, which is a part of the stomach, the is not the best way to prevent the client from
jejunum, and a portion of the stomach are coughing out the tracheotomy. Asking the doctor
removed and anastomosed. Answer: B. is to suture the tracheotomy in place is not
incorrect because the proximal third of the small appropriate.
intestine is not removed. The entire stomach is 13. Answer: D. Notifying the physician
not removed, as in answer C, and in answer D, The output of 300 mL is indicative of
the esophagus is not removed. hemorrhage and should be reported
7. Answer: C. Fresh raw pepper immediately. Answer: A. does nothing to help
Fresh raw or whole pepper are not allowed the client. Milking the tube is done only with an
unless thoroughly cooked in food. Canned fruits order and will not help in this situation, and
are allowed since they are processed and slowing the intravenous infusion is not correct;
pasteurized. Salt, ketchup, and sugar are thus, answers B and C are incorrect.
allowed. 14. Answer: A. Digoxin
8. Answer: A. Have a Protime done monthly The infant with Tetralogy of Fallot involves four
Coumadin is an anticoagulant. One of the tests heart defects: A large ventricular septal defect
for bleeding time is a Protime. This test should (VSD), Pulmonary stenosis, Right ventricular
be done monthly. Eating more fruits and hypertrophy and, An overriding aorta. He will be
vegetables is not necessary, and dark-green treated with digoxin to slow and strengthen the
vegetables contain vitamin K, which increases heart. Epinephrine, aminophylline, and atropine
clotting, so answer B is incorrect. Drinking more will speed the heart rate and are not used in this
liquids and avoiding crowds is not necessary, so client; therefore, answers B, C, and D are
answers C and D are incorrect. incorrect.
9. Answer: A. Perform the Valsalva maneuver 15. Answer: A.
as the catheter is advanced The Tail of Spence is located in the upper outer
quadrant of the breast.
16. Answer: A. Tire easily voiding and bowel movements, so answer D is
The toddler with a ventricular septal defect will incorrect.
tire easily. He will not grow normally but will not 23. Answer: C. Feeding the client with
need more calories. He will be susceptible to dementia
bacterial infection, but he will be no more Of these clients, the one who should be
susceptible to viral infections than other children. assigned to the care of the nursing assistant is
Therefore, answers B, C, and D are incorrect. the client with dementia. Only an RN or the
17. Answer: B. Measure the fetal activity physician can place the client in seclusion, so
A nonstress test determines periodic movement answer A (Placing the client in seclusion) is
of the fetus. It does not determine lung maturity, incorrect. The nurse should empty the Foley
show contractions, or measure neurological catheter of the preeclamptic client because the
well-being, making answers A, C, and D client is unstable, making answer B incorrect. A
incorrect. nurse or physical therapist should ambulate the
18. Answer: C. Turn off the Pitocin infusion client with a fractured hip, so answer D is
The monitor indicates variable decelerations incorrect.
caused by cord compression. If Pitocin is 24. Answer: A. A tracheotomy set
infusing, the nurse should turn off the Pitocin. The client who has recently had a thyroidectomy
Instructing the client to push is incorrect is at risk for tracheal edema. A padded tongue
because pushing could increase the blade is used for seizures and not for the client
decelerations and because the client is 8cm with tracheal edema, so answer B is incorrect. If
dilated, making answer A incorrect. Performing a the client experiences tracheal edema, the
vaginal exam should be done after turning off endotracheal tube or airway will not correct the
the Pitocin, and placing the client in a semi- problem, so answers C and D are incorrect.
Fowlers position is not appropriate for this 25. Answer: D. Birds
situation; therefore, answers B and D are Histoplasmosis is a fungus carried by birds. It is
incorrect. not transmitted to humans by cats, dogs, or
19. Answer: C. Ventricular tachycardia turtles. Therefore, answers A, B, and C are
The graph indicates ventricular tachycardia. The incorrect.
answers in A, B, and D are not noted on the 26. Answer: B. Administer oxygen.
ECG strip. Administering supplemental oxygen to the
20. Answer: B. Be injected into the abdomen patient is the first priority. Administer oxygen to
Lovenox injections should be given in the increase SpO2 to greater than 90% to help
abdomen, not in the deltoid muscle. The client prevent further cardiac damage. Sublingual
should not aspirate after the injection or clear nitroglycerin and morphine are commonly
the air from the syringe before injection. administered after oxygen.
Therefore, answers A, C, and D are incorrect. 27. Answer: D. Lower back pain.
21. Answer: B. Administer the medication Lower back pain results from expansion of the
separately aneurysm. The expansion applies pressure in
Valium is not given in the same syringe with the abdomen, and the pain is referred to the
other medications, so answer A (Douche after lower back.
intercourse) is incorrect. These medications can Option A: Abdominal pain is the most
be given to the same client, so answer D common symptom resulting from
(Ambulating the client with a fractured hip) is impaired circulation.
incorrect. In answer C (Feeding the client with Option B: Absent pedal pulses are a
dementia), it is not necessary to wait to inject the sign of no circulation and would occur
second medication. Valium is an antianxiety after a ruptured aneurysm or in
medication, and Phenergan is used as an peripheral vascular disease.
antiemetic. Option C: Chest pain usually is
22. Answer: B. Void every 3 hours associated with coronary artery or
Voiding every 3 hours prevents stagnant urine pulmonary disease.
from collecting in the bladder, where bacteria 28. Answer: B. Hypertrophic
can grow. Douching is not recommended and Cardiac output isnt affected by hypertrophic
obtaining a urinalysis monthly is not necessary, cardiomyopathy because the size of the
making answers A and C incorrect. The client ventricle remains relatively unchanged. Dilated
should practice wiping from front to back after cardiomyopathy, obliterative cardiomyopathy,
and restrictive cardiomyopathy all decrease Showing this patient how to splint his chest wall
cardiac output. will help decrease discomfort when coughing.
29. Answer: A. Have the patient sit down. Option A: Holding in his coughs will only
The initial priority is to decrease oxygen increase his pain.
consumption by sitting the patient down. Option B: Placing the head of the bed
Administer sublingual nitroglycerin (Option D) as flat may increase the frequency of his
you simultaneously do the ECG. When the cough and his work of breathing.
patients condition is stabilized, he can be Option C: Increasing fluid intake will
returned to bed (Option B). help thin the secretions, making it easier
30. Answer: C. High Fowlers position. for him to clear them.
High Fowlers position facilitates breathing by 36. Answer: B. Give a bronchodilator by
reducing venous return. Lying flat and side-lying nebulizer.
positions worsen breathing and increase the The patient having an acute asthma attack
hearts workload. needs more oxygen delivered to his lungs and
31. Answer: C. Disseminated intravascular body. Nebulized bronchodilators open airways
coagulation (DIC). and increase the amount of oxygen delivered.
Abruptio placentae is a cause of DIC because it Option C: The patient may not need
activates the clotting cascade after hemorrhage. cardiac monitoring because hes only 19
Option A: Thrombocytopenia results years old, unless he has a medical
from decreased production of platelets. history of cardiac problems.
Option B: ITP doesnt have a definitive Options A and D: Important but not
cause. priority as of the moment; emotional
Option D: A patient with abruptio support can help calm the patient but
placentae wouldnt get heparin and, as a can be done after medical intervention.
result, wouldnt be at risk for HATT. 37. Answer: A. Acute respiratory distress
32. Answer: D. Packed red blood cells. syndrome (ARDS).
In a trauma situation, the first blood product Severe hypoxia after smoke inhalation typically
given is unmatched (O negative) packed red is related to ARDS. The other choices arent
blood cells. typically associated with smoke inhalation.
Option A: Fresh frozen plasma often is 38. Answer: D. The water-seal chamber
used to replace clotting factors. doesnt fluctuate when no suction is applied.
Options B and C: Lactated Ringers The chest tube isnt removed until the patients
solution or 0.9% sodium chloride is used lung has adequately re-expanded and is
to increase volume and blood pressure, expected to stay that way. One indication of
but too much of these crystalloids will reexpansion is the cessation of fluctuation in the
dilute the blood and wont improve water-seal chamber when suction isnt applied.
oxygen-carrying capacity. Option C: The chest X-ray should show
33. Answer: C. Immune response. that the lung is re-expanded.
Corticosteroids suppress eosinophils, Option A: Drainage should be minimal
lymphocytes, and natural-killer cells, inhibiting before the chest tube is removed.
the natural inflammatory process in an infected Option B: An ABG test isnt necessary if
or injured part of the body. This helps resolve clinical assessment criteria are met.
inflammation, stabilizes lysosomal membranes, 39. Answer: A. High-top sneakers.
decreases capillary permeability, and depresses High-top sneakers are used to prevent foot drop
phagocytosis of tissues by white blood cells, and contractures in patients with neurologic
thus blocking the release of more inflammatory conditions. Option C: A consult with physical
materials. Excessive corticosteroid therapy can therapy is important to prevent foot drop, but you
lead to Cushings syndrome. can use high-top sneakers independently.
34. Answer: B. It interferes with viral 40. Answer: C. Restlessness and confusion.
replication. The earliest sign of increased ICP is a change in
Zidovudine inhibits DNA synthesis in HIV, thus mental status.
interfering with viral replication. The drug doesnt Option A: Bradycardia and widened
destroy the viral wall, stimulate the immune pulse pressure occur later.
system, or promote HIV antibody excretion. Option B: The patient may void a lot of
35. Answer: D. Splint your chest wall with a very dilute urine if his posterior pituitary
pillow for comfort. is damaged.
41. Answer: C. Mix the drug with saline During periods of infection or illness, patients
solution only. with Type 1 diabetes may need even more
Phenytoin is compatible only with saline insulin to compensate for increased blood
solutions; dextrose causes an insoluble glucose levels.
precipitate to form. You neednt withhold 48. Answer: A. Hematoma.
additional anticonvulsants or use an in-line filter A decreased hematocrit level is a sign of
(Option B.). hematoma, a delayed complication of abdominal
42. Answer: A. Abduction and vaginal hysterectomy.
After surgical repair of the hip, keep the legs and Option B: Symptoms of hypovolemia
hips abducted to stabilize the prosthesis in the include increased hematocrit and
acetabulum. hemoglobin values.
43. Answer: B. Fluid and electrolyte balance. Option C: Infection manifests with fever
Acute pancreatitis is commonly associated with and high WBC count.
fluid isolation and accumulation in the bowel Option D: Symptoms of a PE include
secondary to ileus or peripancreatic dyspnea, chest pain, cough,
edema. Fluid and electrolyte loss from vomiting hemoptysis, restlessness, and signs of
is a major concern. Therefore, your priority is to shock.
manage hypovolemia and restore electrolyte 49. Answer: C. Lactated Ringers solution.
balance. Lactated Ringers solution replaces lost sodium
Options A & D: Pain control and nutrition and corrects metabolic acidosis, both of which
also are important, but not priority. commonly occur following a burn.
Option C: Patients are at risk for Option A: Albumin is used as adjunct
hyperglycemia, not hypoglycemia. therapy, not primary fluid replacement.
44. Answer: B. Right side-lying, with the bed Option B: Dextrose isnt given to burn
flat. patients during the first 24 hours
Positioning the patient on his right side with the because it can cause pseudodiabetes.
bed flat will splint the biopsy site and minimize Option D: The patient is hyperkalemic
bleeding. The other positions wont do this and from the potassium shift from the
may cause more bleeding at the site or intracellular space to the plasma, so
internally. potassium would be detrimental.
45. Answer: B. Angina or cardiac arrhythmia. 50. Answer: C. Gently roll a sterile swab from
Precipitation of angina or cardiac arrhythmia is a the center of the wound outward to collect
potentially serious complication of drainage.
hypothyroidism treatment. Rolling a swab from the center outward is the
Option A: Acute hemolytic reaction is a right way to obtain a culture specimen from a
complication of blood transfusions. wound.
Option C: Retinopathy typically is Option A: Irrigating the wound washes
a complication of diabetes mellitus. away drainage, debris, and many of the
Option D: Thrombocytopenia doesnt colonizing or infecting microorganisms.
result from treating hypothyroidism. Option B: The outside of the wound and the
46. Answer: B. Diabetes insipidus. dressing may be colonized with microorganisms
Maintaining adequate fluid and replacing that havent affected the wound, so specimens
vasopressin are the main objectives in treating from these sites could give inaccurate results.
diabetes insipidus.
Option A: Diabetes is a chronic NCLEX Questions 5
condition associated with abnormally Answers and Rationale
high levels of sugar (glucose) in the 1. Answer: C. Decreased pain.
blood. Insulin produced by the pancreas Furosemide, a loop diuretic, does not alter pain.
lowers blood glucose. Furosemide acts on the kidneys to increase
Option C: Diabetic ketoacidosis is a urinary output. Fluid may move from the
result of severe insulin insufficiency. periphery, decreasing edema. Fluid load is
Option D: An excess of antidiuretic reduced, lowering blood pressure.
hormone leads to SIADH, causing the 2. Answer: A. Obesity.
patient to retain fluid. Obesity is an important risk factor for coronary
47. Answer: C. More insulin. artery disease that can be modified by improved
diet and weight loss. Family history of coronary
artery disease, male gender, and advancing age Three months after surgery and chemotherapy
increase risk but cannot be modified. the patient is likely to be feeling the after-effects,
3. Answer: B. History of cerebral which often includes anemia because of bone-
hemorrhage. marrow suppression. There is no evidence that
A history of cerebral hemorrhage is a the patient is immunosuppressed, and fatigue is
contraindication to tPA because it may increase not a typical symptom of immunosuppression.
the risk of bleeding. TPA acts by dissolving the The information given does not indicate that
clot blocking the coronary artery and works best depression or dehydration is a cause of her
when administered within 6 hours of onset of symptoms.
symptoms. Prior MI is not a contraindication to 9. Answer: C. The patient should use iron
tPA. Patients receiving tPA should be observed cookware to prepare foods, such as dark
for changes in blood pressure, as tPA may green, leafy vegetables and legumes, which
cause hypotension. are high in iron.
4. Answer: C. Prevents DVT (deep vein Normal hemoglobin values range from 11.5-
thrombosis). 15.0. This vegetarian patient is mildly anemic.
Exercise is important for all hospitalized patients When food is prepared in iron cookware its iron
to prevent deep vein thrombosis. Muscular content is increased. In addition, dark green
contraction promotes venous return and leafy vegetables, such as spinach and kale, and
prevents hemostasis in the lower extremities. legumes are high in iron. Mild anemia does not
This exercise is not sufficiently vigorous to require that animal sources of iron be added to
increase physical fitness, nor is it intended to the diet. Many non-animal sources are available.
prevent bedsores or constipation. Coffee and tea increase gastrointestinal activity
5. Answer: D. Confusion. and inhibit absorption of iron.
Cardiogenic shock severely impairs the pumping 10. Answer: D. A nurse should remain in the
function of the heart muscle, causing diminished room during the first 15 minutes of infusion.
blood flow to the organs of the body. This results Transfusion reaction is most likely during the
in diminished brain function and confusion, as first 15 minutes of infusion, and a nurse should
well as hypotension, tachycardia, and weak be present during this period. PRBCs should be
pulse. Cardiogenic shock is a serious infused through a 19g or larger IV catheter to
complication of myocardial infarction with a high avoid slow flow, which can cause clotting.
mortality rate. PRBCs must be flushed with 0.45% normal
6. Answer: D. Check blood pressure. saline solution. Other intravenous solutions will
A patient with congestive heart failure and hemolyze the cells.
dyspnea may have pulmonary edema, which 11. Answer: B. An increase in hematocrit.
can cause severe hypertension. Therefore, Epoetin is a form of erythropoietin, which
taking the patients blood pressure should be the stimulates the production of red blood cells,
first action. Lying flat on the exam table would causing an increase in hematocrit. Epoetin is
likely worsen the dyspnea, and the patient may given to patients who are anemic, often as a
not tolerate it. Blood draws for chemistry and result of chemotherapy treatment. Epoetin has
ABG will be required, but not prior to the blood no effect on neutrophils, platelets, or serum iron.
pressure assessment. 12. Answers: B, C, and D.
7. Answer: C. Headaches are a frequent side Polycythemia vera is a condition in which the
effect of nitroglycerine because it causes bone marrow produces too many red blood
vasodilation. cells. This causes an increase in hematocrit and
Nitroglycerin is a potent vasodilator and often viscosity of the blood. Patients can experience
produces unwanted effects such as headache, headaches, dizziness, and visual disturbances.
dizziness, and hypotension. Patients should be Cardiovascular effects include increased blood
counseled, and the dose titrated, to minimize pressure and delayed clotting time. Weight loss
these effects. In spite of the side effects, is not a manifestation of polycythemia vera.
nitroglycerin is effective at reducing myocardial 13. Answer: A. Observe for evidence of
oxygen consumption and increasing blood flow. spontaneous bleeding.
The patient should not stop the medication. Platelet counts under 30,000/microliter may
Nitroglycerine does not cause bleeding in the cause spontaneous petechiae and bruising,
brain. particularly in the extremities. When the count
8. Answer: A. The symptoms may be the falls below 15,000, spontaneous bleeding into
result of anemia caused by chemotherapy. the brain and internal organs may occur.
Headaches may be a sign and should be A definitive diagnosis of Hodgkins disease is
watched for. Aspirin disables platelets and made if Reed-Sternberg cells are found on
should never be used in the presence of pathologic examination of the excised lymph
thrombocytopenia. Thrombocytopenia does not node. Lymphoblasts are immature cells found in
compromise immunity, and there is no reason to the bone marrow of patients with acute
limit visitors as long as any physical trauma is lymphoblastic leukemia. Gauchers cells are
prevented. large storage cells found in patients with
14. Answers: A, B, and D. Gauchers disease. Rieders cells are
Side effects of corticosteroids include weight myeloblasts found in patients with acute
gain, fluid retention with hypertension, myelogenous leukemia.
Cushingoid features, a low serum albumin, and 20. Answer: C. Stay with the patient and
suppressed inflammatory response. Patients are focus on slow, deep breathing for relaxation.
encouraged to eat a diet high in protein, Slow, deep breathing is the most effective
vitamins, and minerals and low in sodium. method of reducing anxiety and stress. It
Corticosteroids cause hypernatremia and not reduces the level of carbon dioxide in the brain
hyponatremia. to increase calm and relaxation. Warning the
15. Answer: B. Change gloves immediately patient to remain still will likely increase her
after use. anxiety. Encouraging family members to stay
The neutropenic patient is at risk of infection. with the patient may make her worry about their
Changing gloves immediately after use protects anxiety as well as her own. Delaying the
patients from contamination with organisms procedure is unlikely to allay her fears.
picked up on hospital surfaces. This 21. Answer: D. Capillary refill of < 3 seconds
contamination can have serious consequences It is important to assess the extremities for blood
for an immunocompromised patient. Changing vessel occlusion in the client with sickle cell
the respiratory mask is desirable, but not nearly anemia because a change in capillary refill
as urgent as changing gloves. Minimizing would indicate a change in circulation. Body
contact and conversation are not necessary and temperature, motion, and sensation would not
may cause nursing staff to miss changes in the give information regarding peripheral circulation;
patients symptoms or condition. therefore, answers A, B, and C are incorrect.
16. Answer: C. We will bring in fresh flowers 22. Answer: D. Semi-Fowlers with legs
to brighten the room. extended on the bed
During induction chemotherapy, the leukemia Placing the client in semi-Fowlers position
patient is severely immunocompromised and at provides the best oxygenation for this client.
risk of serious infection. Fresh flowers, fruit, and Flexion of the hips and knees, which includes
plants can carry microbes and should be the knee-chest position, impedes circulation and
avoided. Books, pictures, and other personal is not correct positioning for this client.
items can be cleaned with antimicrobials before Therefore, answers A, B, and C are incorrect.
being brought into the room to minimize the risk 23. Answer: B. Encouraging fluid intake of at
of contamination. least 200mL per hour
17. Answer: A. 3-10 years. It is important to keep the client in sickle cell
The peak incidence of ALL is at 4 years (range crisis hydrated to prevent further sickling of the
3-10). It is uncommon after the mid-teen years. blood. Answer A is incorrect because a
The peak incidence of chronic myelogenous mechanical cuff places too much pressure on
leukemia (CML) is 45-55 years. The peak the arm. Answer C is incorrect because raising
incidence of acute myelogenous leukemia (AML) the knee gatch impedes circulation. Answer D is
occurs at 60 years. Two-thirds of cases of incorrect because Tylenol is too mild an
chronic lymphocytic leukemia (CLL) occur after analgesic for the client in crisis.
60 years. 24. Answer: C. Popsicle
18. Answer: B. Night sweats and fatigue. Hydration is important in the client with sickle
Symptoms of Hodgkins disease include night cell disease to prevent thrombus formation.
sweats, fatigue, weakness, and tachycardia. The Popsicles, gelatin, juice, and pudding have high
disease is characterized by painless, enlarged fluid content. The foods in answers A, B, and D
cervical lymph nodes. Weight loss occurs early do not aid in hydration and are, therefore,
in the disease. Nausea and vomiting are not incorrect.
typically symptoms of Hodgkins disease. 25. Answer: C. Start O2
19. Answer: A. Reed-Sternberg cells.
The most prominent clinical manifestation of The client with polycythemia vera is at risk for
sickle cell crisis is pain. However, the pulse thrombus formation. Hydrating the client with at
oximetry indicates that oxygen levels are low; least 3L of fluid per day is important in
thus, oxygenation takes precedence over pain preventing clot formation, so the statement to
relief. Answer A is incorrect because although a drink less than 500mL is incorrect. Answers B,
warm environment reduces pain and minimizes C, and D are incorrect because they all
sickling, it would not be a priority. Answer B is contribute to the prevention of complications.
incorrect because although hydration is Support hose promotes venous return, the
important, it would not require a bolus. Answer D electric razor prevents bleeding due to injury,
is incorrect because Demerol is acidifying to the and a diet low in iron is essential to preventing
blood and increases sickling. further red cell formation.
26. Answer: C. Egg salad on wheat bread, 32. Answer: C. The client had radiation for
carrot sticks, lettuce salad, raisin pie treatment of Hodgkins disease as a
Egg yolks, wheat bread, carrots, raisins, and teenager.
green, leafy vegetables are all high in iron, Radiation treatment for other types of cancer
which is an important mineral for this client. can result in leukemia. Some hobbies and
Roast beef, cabbage, and pork chops are also occupations involving chemicals are linked to
high in iron, but the side dishes accompanying leukemia, but not the ones in these answers;
these choices are not; therefore, answers A, B, therefore, answers A and B are incorrect.
and D are incorrect. Answer D is incorrect because the incidence of
27. Answer: D. A bus trip to the Museum of leukemia is higher in twins than in siblings.
Natural History 33. Answer: D. The soles of the feet
Taking a trip to the museum is the only answer Petechiae are not usually visualized on dark
that does not pose a threat. A family vacation in skin. The soles of the feet and palms of the hand
the Rocky Mountains at high altitudes, cold provide a lighter surface for assessing the client
temperatures, and airplane travel can cause for petechiae. Answers A, B, and C are incorrect
sickling episodes and should be avoided; because the skin might be too dark to make an
therefore, answers A, B, and C are incorrect. assessment.
28. Answer: D. Examine the tongue 34. Answer: B. Have you had a respiratory
The tongue is smooth and beefy red in the client infection in the last 6 months?
with vitamin B12 deficiency, so examining the The client with leukemia is at risk for infection
tongue should be included in the physical and has often had recurrent respiratory
assessment. Bleeding, splenomegaly, and blood infections during the previous 6 months.
pressure changes do not occur, making answers Insomnolence, weight loss, and a decrease in
A, B, and C incorrect. alertness also occur in leukemia, but bleeding
29. Answer: C. Roof of the mouth tendencies and infections are the primary clinical
The oral mucosa and hard palate (roof of the manifestations; therefore, answers A, C, and D
mouth) are the best indicators of jaundice in are incorrect.
dark-skinned persons. The conjunctiva can have 35. Answer: B. Risk for injury related to
normal deposits of fat, which give a yellowish thrombocytopenia
hue; thus, answer A is incorrect. The soles of The client with acute leukemia has bleeding
the feet can be yellow if they are calloused, tendencies due to decreased platelet counts,
making answer B incorrect; the shins would be and any injury would exacerbate the problem.
an area of darker pigment, so answer D is The client would require close monitoring for
incorrect. hemorrhage, which is of higher priority than the
30. Answer: B. Respirations 28 shallow diagnoses in answers A, C, and D, which are
When there are fewer red blood cells, there is incorrect.
less hemoglobin and less oxygen. Therefore, the 36. Answer: A. Sexual dysfunction related to
client is often short of breath, as indicated in radiation therapy
answer B. The client with anemia is often pale in Radiation therapy often causes sterility in male
color, has weight loss, and may be hypotensive. clients and would be of primary importance to
Answers A, C, and D are within normal and, this client. The psychosocial needs of the client
therefore, are incorrect. are important to address in light of the age and
31. Answer: A. I will drink 500mL of fluid or life choices. Hodgkins disease, however, has a
less each day. good prognosis when diagnosed early. Answers
B, C, and D are incorrect because they are of 43. Answer: A. Glucometer readings as
lesser priority. ordered
37. Answer: A. Platelet count IV glucocorticoids raise the glucose levels and
Clients with autoimmune thrombocytopenic often require coverage with insulin. Answer B is
purpura (ATP) have low platelet counts, making not necessary at this time, sodium and
answer A the correct answer. White cell counts, potassium levels would be monitored when the
potassium levels, and PTT are not affected in client is receiving mineralocorticoids, and daily
ATP; thus, answers B, C, and D are incorrect. weights is unnecessary; therefore, answers B,
38. Answer: A. Bleeding precautions C, and D are incorrect.
The normal platelet count is 120,000400, 44. Answer: B. Check the calcium level
Bleeding occurs in clients with low platelets. The The parathyroid glands are responsible for
priority is to prevent and minimize bleeding. calcium production and can be damaged during
Oxygenation in answer C is important, but a thyroidectomy. The tingling is due to low
platelets do not carry oxygen. Answers B and D calcium levels. The crash cart would be needed
are of lesser priority and are incorrect in this in respiratory distress but would not be the next
instance. action to take; thus, answer A is incorrect.
39. Answer: C. Elevate the head of the bed Hypertension occurs in thyroid storm and the
30 drainage would occur in hemorrhage, so
Elevating the head of the bed 30 avoids answers C and D are incorrect.
pressure on the sella turcica and alleviates 45. Answer: D. Decreased cardiac output r/t
headaches. Answers A, B, and D are incorrect bradycardia
because Trendelenburg, Valsalva maneuver, The decrease in pulse can affect the cardiac
and coughing all increase the intracranial output and lead to shock, which would take
pressure. precedence over the other choices; therefore,
40. Answer: B. Check the vital signs answers A, B, and C are incorrect.
The large amount of fluid loss can cause fluid
and electrolyte imbalance that should be NCLEX Questions 6
corrected. The loss of electrolytes would be Answers and Rationale
reflected in the vital signs. Measuring the urinary 1. Answer: B. That feeling of warmth is
output is important, but the stem already says normal when the dye is injected.
that the client has polyuria, so answer A is It is normal for the client to have a warm
incorrect. Encouraging fluid intake will not sensation when dye is injected. Answers A, C,
correct the problem, making answer C incorrect. and D indicate that the nurse believes that the
Answer D is incorrect because weighing the hot feeling is abnormal, so they are incorrect.
client is not necessary at this time. 2. Answer: D. The nurse wears gloves to take
41. Answer: C. Pinch the soft lower part of the clients vital signs.
the nose for a minimum of 5 minutes It is not necessary to wear gloves to take the
The client should be positioned upright and vital signs of the client. If the client has active
leaning forward, to prevent aspiration of blood. infection with methicillin-resistant
Answers A, B, and D are incorrect because staphylococcus aureus, gloves should be worn.
direct pressure to the nose stops the bleeding, The health care workers in answers A, B, and C
and ice packs should be applied directly to the indicate knowledge of infection control by their
nose as well. If a pack is necessary, the nares actions.
are loosely packed. 3. Answer: D. The client has a grand mal
42. Answer: A. Blood pressure seizure.
Blood pressure is the best indicator of During ECT, the client will have a grand mal
cardiovascular collapse in the client who has seizure. This indicates completion of the
had an adrenal gland removed. The remaining electroconvulsive therapy. Answers A, B, and C
gland might have been suppressed due to the do not indicate that the ECT has been effective,
tumor activity. Temperature would be an so are incorrect.
indicator of infection, decreased output would be 4. Answer: A. Examine the perianal area with
a clinical manifestation but would take longer to a flashlight 2 or 3 hours after the child is
occur than blood pressure changes, and specific asleep
gravity changes occur with other disorders; Infection with pinworms begins when the eggs
therefore, answers B, C, and D are incorrect. are ingested or inhaled. The eggs hatch in the
upper intestine and mature in 28 weeks. The
females then mate and migrate out the anus, act committed on the client or their belongings;
where they lay up to 17,000 eggs. This causes and assault is a violent physical or verbal attack.
intense itching. The mother should be told to use 9. Answer: D. Starting a blood transfusion
a flashlight to examine the rectal area about 23 The licensed practical nurse should not be
hours after the child is asleep. Placing clear tape assigned to begin a blood transfusion. The
on a tongue blade will allow the eggs to adhere licensed practical nurse can insert a Foley
to the tape. The specimen should then be catheter, discontinue a nasogastric tube, and
brought in to be evaluated. There is no need to collect sputum specimen; therefore, answers A,
scrape the skin, collect a stool specimen, or B, and C are incorrect.
bring a sample of hair, so answers B, C, and D 10. Answer: B. Contacting the physician
are incorrect. The vital signs are abnormal and should be
5. Answer: B. The entire family should be reported immediately. Continuing to monitor the
treated. vital signs can result in deterioration of the
Enterobiasis, or pinworms, is treated with clients condition, making answer A incorrect.
Vermox (mebendazole) or Antiminth (pyrantel Asking the client how he feels in answer C will
pamoate). The entire family should be treated to only provide subjective data, and the nurse in
ensure that no eggs remain. Because a single answer D is not the best nurse to assign
treatment is usually sufficient, there is usually because this client is unstable.
good compliance. The family should then be 11. Answer: B. The RN with 3 years of
tested again in 2 weeks to ensure that no eggs experience in labor and delivery
remain. Answers A, C, and D are incorrect The nurse with 3 years of experience in labor
statements. and delivery knows the most about possible
6. Answer: A. The client receiving linear complications involving preeclampsia. The nurse
accelerator radiation therapy for lung cancer in answer A is a new nurse to the unit, and the
The pregnant nurse should not be assigned to nurses in answers C and D have no experience
any client with radioactivity present. The client with the postpartum client.
receiving linear accelerator therapy travels to the 12. Answer: B. The narcotic count has been
radium department for therapy. The radiation incorrect on the unit for the past 3 days.
stays in the department, so the client is not The Joint Commission on Accreditation of
radioactive. The clients in answers B, C, and D Hospitals will probably be interested in the
pose a risk to the pregnant nurse. These clients problems in answers A and C. The failure of the
are radioactive in very small doses, especially nursing assistant to care for the client with
upon returning from the procedures. For hepatitis might result in termination, but is not of
approximately 72 hours, the clients should interest to the Joint Commission.
dispose of urine and feces in special containers 13. Answer: B. File a formal reprimand
and use plastic spoons and forks. The next action after discussing the problem
7. Answer: A. The client with Cushings with the nurse is to document the incident by
disease filing a formal reprimand. If the behavior
The client with Cushings disease has continues or if harm has resulted to the client,
adrenocortical hypersecretion. This increase in the nurse may be terminated and reported to the
the level of cortisone causes the client to be Board of Nursing, but these are not the first
immunosuppressed. In answer B, the client with actions requested in the stem. A tort is a
diabetes poses no risk to other clients. The wrongful act to the client or his belongings and is
client in answer C has an increase in growth not indicated in this instance. Therefore,
hormone and poses no risk to himself or others. Answers A, C, and D are incorrect.
The client in answer D has hypothyroidism or 14. Answer: D. The 30-year-old with an
myxedema and poses no risk to others or exacerbation of multiple sclerosis being
himself. treated with cortisone via a centrally placed
8. Answer: D. Malpractice venous catheter
The nurse could be charged with malpractice, The client at highest risk for complications is the
which is failing to perform, or performing an act client with multiple sclerosis who is being treated
that causes harm to the client. Giving the infant with cortisone via the central line. The others are
an overdose falls into this category. Answers A, more stable. MRSA is methicillin-resistant
B, and C are incorrect because they apply to staphylococcus aureus. Vancomycin is the drug
other wrongful acts. Negligence is failing to of choice and is given at scheduled times to
perform care for the client; a tort is a wrongful maintain blood levels of the drug. The clients in
answers A, B, and C are more stable and can be Always remember your ABCs (airway, breathing,
seen later. circulation) when selecting an answer. Although
15. Answer: B. The client who is 6 months answers B and D might be appropriate for this
pregnant with abdominal pain and the client child, answer C should have the highest priority.
with facial lacerations and a broken arm Answer A does not apply for a child who has
The pregnant client and the client with a broken undergone a tonsillectomy.
arm and facial lacerations are the best choices 21. Answer: A. High fever
for placing in the same room. The clients in If the child has bacterial pneumonia, a high fever
answers A, C, and D need to be placed in is usually present. Bacterial pneumonia usually
separate rooms due to the serious natures of presents with a productive cough, not a
their injuries. nonproductive cough, making answer B
16. Answer: A. The eye should be cleansed incorrect. Rhinitis is often seen with viral
with warm water, removing any exudate, pneumonia, and vomiting and diarrhea are
before instilling the eyedrops. usually not seen with pneumonia, so answers C
Before instilling eye drops, the nurse should and D are incorrect.
cleanse the area with water. A 6-year-old child is 22. Answer: B. A tracheostomy set
not developmentally ready to instill his own For a child with epiglottitis and the possibility of
eyedrops, so answer B is incorrect. Although the complete obstruction of the airway, emergency
mother of the child can instill the eyedrops, the tracheostomy equipment should always be kept
area must be cleansed before administration, at the bedside. Intravenous supplies, fluid, and
making answer C incorrect. Although the eye oxygen will not treat an obstruction; therefore,
might appear to be clear, the nurse should instill answers A, C, and D are incorrect.
the eyedrops, as ordered, so answer D is 23. Answer: C. Exophthalmos
incorrect. Exophthalmos (protrusion of eyeballs) often
17. Answer: C. We are going on a camping occurs with hyperthyroidism. The client with
trip this weekend, and I have bought hot hyperthyroidism will often exhibit tachycardia,
dogs to grill for his lunch. increased appetite, and weight loss; therefore,
Remember the ABCs (airway, breathing, answers A, B, and D are incorrect.
circulation) when answering this question. 24. Answer: D. Cheese omelet
Answer C because a hotdog is the size and The child with celiac disease should be on a
shape of the childs trachea and poses a risk of gluten-free diet. Answers A, B, and C all contain
aspiration. Answers A, B, and C are incorrect gluten, while answer D gives the only choice of
because white grape juice, a grilled cheese foods that does not contain gluten.
sandwich, and ice cream do not pose a risk of 25. Answer: C. Apply oxygen by mask
aspiration for a child. Remember the ABCs (airway, breathing,
18. Answer: C. Ask the parent/guardian to circulation) when answering this question.
room-in with the child. Before notifying the physician or assessing the
The nurse should encourage rooming-in to pulse, oxygen should be applied to increase the
promote parent-child attachment. It is okay for oxygen saturation, so answers A and D are
the parents to be in the room for assessment of incorrect. The normal oxygen saturation for a
the child. Allowing the child to have items that child is 92%100%, making answer B incorrect.
are familiar to him is allowed and encouraged; 26. Answer: B. A moderate amount of straw-
therefore, answers A and B are incorrect. colored fluid
Answer D is not part of the nurses An amniotomy is an artificial rupture of
responsibilities. membranes and normal amniotic fluid is straw-
19. Answer: B. Store the hearing aid in a colored and odorless. Fetal heart tones of 160
warm place. indicate tachycardia, and greenish fluid is
The hearing aid should be stored in a warm, dry indicative of meconium, so answers A and C are
place. It should be cleaned daily but should not incorrect. If the nurse notes the umbilical cord,
be moldy, so answer A is incorrect. A toothpick the client is experiencing a prolapsed cord, so
is inappropriate to use to clean the aid; the answer D is incorrect and would need to be
toothpick might break off in the hearing aide, reported immediately.
making answer C incorrect. Changing the 27. Answer: D. When can I get my
batteries weekly, as in answer D, is not epidural?
necessary. Dilation of 2 cm marks the end of the latent
20. Answer: C. Risk for aspiration phase of labor. Answer A is a vague answer,
answer B indicates the end of the first stage of The initial action by the nurse observing a late
labor, and answer C indicates the transition deceleration should turn the client to the side
phase. preferably, the left side. Administering oxygen is
28. Answer: B. Turn the client to her left side also indicated. Answer A might be necessary but
The normal fetal heart rate is 120160 bpm; not before turning the client to her side. Answer
100110bpm is bradycardia. The first action B is not necessary at this time. Answer D is
would be to turn the client to the left side and incorrect because there is no data to indicate
apply oxygen. Answer A is not indicated at this that the monitor has been applied incorrectly.
time. Answer C is not the best action for clients 35. Answer: D. Acceleration of FHR with fetal
experiencing bradycardia. There is no data to movements
indicate the need to move the client to the Accelerations with movement are normal.
delivery room at this time. Answers A, B, and C indicate ominous findings
29. Answer: D. Progressive cervical dilation on the fetal heart monitor.
The expected effect of Pitocin is cervical dilation. 36. Answer: C. The sensation of the bladder
Pitocin causes more intense contractions, which filling is diminished or lost.
can increase the pain, making answer A Epidural anesthesia decreases the urge to void
incorrect. Cervical effacement is caused by and sensation of a full bladder. A full bladder will
pressure on the presenting part, so answer B is decrease the progression of labor. Answers A,
incorrect. Answer C is opposite the action of B, and D are incorrect for the stem.
Pitocin. 37. Answer: B. Luteinizing hormone is high.
30. Answer: B. Apply the fetal heart monitor Luteinizing hormone released by the pituitary is
Applying a fetal heart monitor is the correct responsible for ovulation. At about day 14, the
action at this time. There is no need to prepare continued increase in estrogen stimulates the
for a Caesarean section or to place the client in release of luteinizing hormone from the anterior
Genupectoral position (knee-chest), so answers pituitary. The LH surge is responsible for
A and C are incorrect. Answer D is incorrect ovulation, or the release of the dominant follicle
because there is no need for an ultrasound in preparation for conception, which occurs
based on the finding. within the next 1012 hours after the LH levels
31. Answer: B. The membranes are still peak. Answers A, C, and D are incorrect
intact. because estrogen levels are high at the
The nurse decides to apply an external monitor beginning of ovulation, the endometrial lining is
because the membranes are intact. Answers A, thick, not thin, and the progesterone levels are
C, and D are incorrect. The cervix is dilated high, not low.
enough to use an internal monitor, if necessary. 38. Answer: C. Regularity of the menses
An internal monitor can be applied if the client is The success of the rhythm method of birth
at 0-station. Contraction intensity has no bearing control is dependent on the clients menses
on the application of the fetal monitor. being regular. It is not dependent on the age of
32. Answer: D. Potential fluid volume deficit the client, frequency of intercourse, or range of
related to decreased fluid intake the clients temperature; therefore, answers A,
Clients admitted in labor are told not to eat B, and D are incorrect.
during labor, to avoid nausea and vomiting. Ice 39. Answer: C. Diaphragm
chips may be allowed, but this amount of fluid The best method of birth control for the client
might not be sufficient to prevent fluid volume with diabetes is the diaphragm. A permanent
deficit. In answer A, impaired gas exchange intrauterine device can cause a continuing
related to hyperventilation would be indicated inflammatory response in diabetics that should
during the transition phase. Answers B and C be avoided, oral contraceptives tend to elevate
are not correct in relation to the stem. blood glucose levels, and contraceptive sponges
33. Answer: D. There is uteroplacental are not good at preventing pregnancy.
insufficiency. Therefore, answers A, B, and D are incorrect.
This information indicates a late deceleration. 40. Answer: D. Sudden, stabbing pain in the
This type of deceleration is caused by lower quadrant
uteroplacental lack of oxygen. Answer A has no The signs of an ectopic pregnancy are vague
relation to the readings, so its incorrect; answer until the fallopian tube ruptures. The client will
B results in a variable deceleration; and answer complain of sudden, stabbing pain in the lower
C is indicative of an early deceleration. quadrant that radiates down the leg or up into
34. Answer: C. Reposition the client the chest. Painless vaginal bleeding is a sign of
placenta previa, abdominal cramping is a sign of misses two, she should take two; if she misses
labor, and throbbing pain in the upper quadrant more than two, she should take the missed pills
is not a sign of an ectopic pregnancy, making but use another method of birth control for the
answers A, B, and C incorrect. remainder of the cycle. Answer D is incorrect
41. Answer: C. Baked chicken, fruit cup, because changes in menstrual flow are
potato salad, coleslaw, yogurt, and iced tea expected in clients using oral contraceptives.
All of the choices are tasty, but the pregnant Often these clients have lighter menses.
client needs a diet that is balanced and has 46. Answer: B. Positive HIV
increased amounts of calcium. Answer A is Clients with HIV should not breastfeed because
lacking in fruits and milk. Answer B contains the the infection can be transmitted to the baby
potato chips, which contain a large amount of through breast milk. The clients in answers A, C,
sodium. Answer C contains meat, fruit, potato and Dthose with diabetes, hypertension, and
salad, and yogurt, which has about 360 mg of thyroid diseasecan be allowed to breastfeed.
calcium. Answer D is not the best diet because it 47. Answer: A. Assess the fetal heart tones
lacks vegetables and milk products. The symptoms of painless vaginal bleeding are
42. Answer: B. Metabolic acidosis with consistent with placenta previa. Answers B, C,
dehydration and D are incorrect. Cervical check for dilation is
The client with hyperemesis has persistent contraindicated because this can increase the
nausea and vomiting. With vomiting comes bleeding. Checking for firmness of the uterus
dehydration. When the client is dehydrated, she can be done, but the first action should be to
will have metabolic acidosis. Answers A and C check the fetal heart tones. A detailed history
are incorrect because they are respiratory can be done later.
dehydration. Answer D is incorrect because the 48. Answer: D. Her contractions are 5
client will not be in alkalosis with persistent minutes apart.
vomiting. The client should be advised to come to the
43. Answer: B. The presence of fetal heart labor and delivery unit when the contractions are
tones every 5 minutes and consistent. She should also
The most definitive diagnosis of pregnancy is be told to report to the hospital if she
the presence of fetal heart tones. The signs in experiences rupture of membranes or extreme
answers A, C, and D are subjective and might bleeding. She should not wait until the
be related to other medical conditions. Answers contractions are every 2 minutes or until she has
A and C may be related to a hydatidiform mole, bloody discharge, so answers A and B are
and answer D is often present before menses or incorrect. Answer C is a vague answer and can
with the use of oral contraceptives. be related to a urinary tract infection.
44. Answer: C. Hypoglycemic, large for 49. Answer: A. Low birth weight
gestational age Infants of mothers who smoke are often low in
The infant of a diabetic mother is usually large birth weight. Infants who are large for gestational
for gestational age. After birth, glucose levels fall age are associated with diabetic mothers, so
rapidly due to the absence of glucose from the answer B is incorrect. Preterm births are
mother. Answer A is incorrect because the infant associated with smoking, but not with
will not be small for gestational age. Answer B is appropriate size for gestation, making answer C
incorrect because the infant will not be incorrect. Growth retardation is associated with
hyperglycemic. Answer D is incorrect because smoking, but this does not affect the infant
the infant will be large, not small, and will be length; therefore, answer D is incorrect.
hypoglycemic, not hyperglycemic. 50. Answer: A. Within 72 hours of delivery
45. Answer: B. An alternate method of birth To provide protection against antibody
control is needed when taking antibiotics. production, RhoGam should be given within 72
When the client is taking oral contraceptives and hours. The answers in B, C, and D are too late
begins antibiotics, another method of birth to provide antibody protection. RhoGam can
control should be used. Antibiotics decrease the also be given during pregnancy.
effectiveness of oral contraceptives.
Approximately 510 pounds of weight gain is not NCLEX Questions 7
unusual, so answer A is incorrect. If the client Answers and Rationale
misses a birth control pill, she should be 1. Answer: B. Fetal heart tones
instructed to take the pill as soon as she When the membranes rupture, there is often a
remembers the pill. Answer C is incorrect. If she transient drop in the fetal heart tones. The heart
tones should return to baseline quickly. Any indicative of compartment syndrome or
alteration in fetal heart tones, such as peripheral vascular disease.
bradycardia or tachycardia, should be reported. 7. Answer: B. Hormonal disturbances
After the fetal heart tones are assessed, the After menopause, women lack hormones
nurse should evaluate the cervical dilation, vital necessary to absorb and utilize calcium. Doing
signs, and level of discomfort, making answers weight-bearing exercises and taking calcium
A, C, and D incorrect. supplements can help to prevent osteoporosis
2. Answer: A. Active but are not causes, so answers A and C are
The active phase of labor occurs when the client incorrect. Body types that frequently experience
is dilated 47cm. The latent or early phase of osteoporosis are thin Caucasian females, but
labor is from 1cm to 3cm in dilation, so answers they are not most likely related to osteoporosis,
B and D are incorrect. The transition phase of so answer D is incorrect.
labor is 810cm in dilation, making answer C 8. Answer: B. The buttocks are 15 off the
incorrect. bed.
3. Answer: B. Wrapping the newborn snugly The infants hips should be off the bed
in a blanket approximately 15 in Bryants traction. Answer A
The infant of an addicted mother will undergo is incorrect because this does not indicate that
withdrawal. Snugly wrapping the infant in a the traction is working correctly, nor does C.
blanket will help prevent the muscle irritability Answer D is incorrect because Bryants traction
that these babies often experience. Teaching is a skin traction, not a skeletal traction.
the mother to provide tactile stimulation or 9. Answer A. Utilizes a Steinman pin
provide for early infant stimulation are incorrect Balanced skeletal traction uses pins and screws.
because he is irritable and needs quiet and little A Steinman pin goes through large bones and is
stimulation at this time, so answers A and D are used to stabilize large bones such as the femur.
incorrect. Placing the infant in an infant seat in Answer B is incorrect because only the affected
answer C is incorrect because this will also leg is in traction. Kirschner wires are used to
cause movement that can increase muscle stabilize small bones such as fingers and toes,
irritability. as in answer C. Answer D is incorrect because
4. Answer: C. Checking the clients blood this type of traction is not used for fractured hips.
pressure 10. Answer A. Serum collection (Davol) drain
Following epidural anesthesia, the client should Bleeding is a common complication of
be checked for hypotension and signs of shock orthopedic surgery. The blood-collection device
every 5 minutes for 15 minutes. The client can should be checked frequently to ensure that the
be checked for cervical dilation later after she is client is not hemorrhaging. The clients pain
stable. The client should not be positioned should be assessed, but this is not life-
supine because the anesthesia can move above threatening. When the client is in less danger,
the respiratory center and the client can stop the nutritional status should be assessed and an
breathing. Fetal heart tones should be assessed immobilizer is not used; thus, answers B, C, and
after the blood pressure is checked. Therefore, D are incorrect.
answers A, B, and D are incorrect. 11. Answer A. I must flush the tube with
5. Answer: B. Wash her hands for 2 minutes water after feedings and clamp the tube.
before care The clients family member should be taught to
The best way to prevent post-operative wound flush the tube after each feeding and clamp the
infection is hand washing. Use of prescribed tube. The placement should be checked before
antibiotics will treat infection, not prevent feedings, and indigestion can occur with the
infections, making answer A incorrect. Wearing PEG tube, just as it can occur with any client, so
a mask and asking the client to cover her mouth answers B and C are incorrect. Medications can
are good practices but will not prevent wound be ordered for indigestion, but it is not a reason
infections; therefore, answers C and D are for alarm. A percutaneous endoscopy
incorrect. gastrostomy tube is used for clients who have
6. Answer: B. Disalignment experienced difficulty swallowing. The tube is
The client with a hip fracture will most likely have inserted directly into the stomach and does not
misalignment. Answers A, C, and D are incorrect require swallowing; therefore, answer D is
because all fractures cause pain, and coolness incorrect.
of the extremities and absence of pulses are 12. Answer C. The clients hematocrit is 26%.
The client with a total knee replacement should suppressed, the 6-year-old with osteomyelitis is
be assessed for anemia. A hematocrit of 26% is infected, and the client in answer A is too old
extremely low and might require a blood and is female; therefore, answers A, C, and D
transfusion. Bleeding of 2cm on the dressing is are incorrect.
not extreme. Circle and date and time the 17. Answer: B. Report chest pain.
bleeding and monitor for changes in the clients Cox II inhibitors have been associated with heart
status. A low-grade temperature is not unusual attacks and strokes. Any changes in cardiac
after surgery. Ensure that the client is well status or signs of a stroke should be reported
hydrated, and recheck the temperature in 1 immediately, along with any changes in bowel or
hour. If the temperature is above 100.6F bladder habits because bleeding has been
(38.1C), report this finding to the doctor. Tylenol linked to use of Cox II inhibitors. The client does
will probably be ordered. Voiding after surgery is not have to take the medication with milk,
also not uncommon and no need for concern; remain upright, or allow 6 weeks for optimal
therefore answers A, B, and D are incorrect. effect, so answers A, C, and D are incorrect.
13. Answer B. The clients parents are skilled 18. Answer: D. Allows 24 hours before
stained-glass artists. bearing weight
Plumbism is lead poisoning. One factor A plaster-of-Paris cast takes 24 hours to dry,
associated with the consumption of lead is and the client should not bear weight for 24
eating from pottery made in Central America or hours. The cast should be handled with the
Mexico that is unfired. The child lives in a house palms, not the fingertips, so answer A is
built after 1976 (this is when lead was taken out incorrect. Petaling a cast is covering the end of
of paint), and the parents make stained glass as the cast with cast batting or a sock, to prevent
a hobby. Stained glass is put together with lead, skin irritation and flaking of the skin under the
which can drop on the work area, where the cast, making answer B incorrect. The client
child can consume the lead beads. Answer A is should be told not to dry the cast with a hair
incorrect because simply traveling out of the dryer because this causes hot spots and could
country does not increase the risk. In answer C, burn the client. This also causes unequal drying;
the house was built after the lead was removed thus, answer C is incorrect.
with the paint. Answer D is unrelated to the 19. Answer: A. It will be alright for your
stem. friends to autograph the cast.
14. Answer: A. High-seat commode There is no reason that the clients friends
The equipment that can help with activities of should not be allowed to autograph the cast; it
daily living is the high-seat commode. The hip will not harm the cast in any way, so answers B,
should be kept higher than the knee. The C, and D are incorrect.
recliner is good because it prevents 90 flexion 20. Answer: A. Assisting the LPN with
but not daily activities. A TENS (Transcutaneous opening sterile packages and peroxide
Electrical Nerve Stimulation) unit helps with pain The nurse is performing the pin care correctly
management and an abduction pillow is used to when she uses sterile gloves and Q-tips. A
prevent adduction of the hip and possibly licensed practical nurse can perform pin care,
dislocation of the prosthesis; therefore, answers there is no need to clean the weights, and the
B, C, and D are incorrect. nurse can help with opening the packages but it
15. Answer: B. Have narcan (naloxone) isnt required; therefore, answers B, C, and D
available are incorrect.
Narcan is the antidote for narcotic overdose. If 21. Answer: A. Check the bowel sounds
hypoxia occurs, the client should have oxygen A body cast or spica cast extends from the
administered by mask, not cannula. There is no upper abdomen to the knees or below. Bowel
data to support the administration of blood sounds should be checked to ensure that the
products or cardiac resuscitation, so answers A, client is not experiencing a paralytic ileus.
C, and D are incorrect. Checking the blood pressure is a treatment for
16. Answer: B. 12-year-old male with a any client, offering pain medication is not called
fractured femur for, and checking for swelling isnt specific to the
The 6-year-old should have a roommate as stem, so answers B, C, and D are incorrect.
close to the same age as possible, so the 12- 22. Answer: C. Halo traction
year-old is the best match. The 10-year-old with Halo traction will be ordered for the client with a
sarcoma has cancer and will be treated with cervical fracture. Russells traction is used for
chemotherapy that makes him immune bones of the lower extremities, as is Bucks
traction. Crutchfield tongs are used while in the Hyperstat is given IV push for hypertensive
hospital and the client is immobile; therefore, crises, but it often causes hyperglycemia. The
answers A, B, and D are incorrect. glucose level will drop rapidly when stopped.
23. Answer: B. The CPM machine controls Answer A is incorrect because the hyperstat is
should be positioned distal to the site. given by IV push. The client should be placed in
The controller for the continuous passive-motion dorsal recumbent position, not a Trendelenburg
device should be placed away from the client. position, as stated in answer C. Answer D is
Many clients complain of pain while having incorrect because the medication does not have
treatments with the CPM, so they might turn off to be covered with foil.
the machine. The CPM flexes and extends the 3. Answer: C. Heart rate of 60 bpm
leg. The client is in the bed during CPM therapy, A heart rate of 60 in the baby should be reported
so answer A is incorrect. Answer C is incorrect immediately. The dose should be held if the
because clients will experience pain with the heart rate is below 100 bpm. The blood glucose,
treatment. Use of the CPM does not alleviate the blood pressure, and respirations are within
need for physical therapy, as suggested in normal limits; thus answers A, B, and D are
answer D. incorrect.
24. Answer: A. Palms rest lightly on the 4. Answer: C. Leave the medication in the
handles brown bottle
The clients palms should rest lightly on the Nitroglycerine should be kept in a brown bottle
handles. The elbows should be flexed no more (or even a special air- and water-tight, solid or
than 30 but should not be extended. Answer B plated silver or gold container) because of its
is incorrect because 0 is not a relaxed angle for instability and tendency to become less potent
the elbows and will not facilitate correct walker when exposed to air, light, or water. The supply
use. The client should walk to the middle of the should be replenished every 6 months, not 3
walker, not to the front of the walker, making months, and one tablet should be taken every 5
answer C incorrect. The client should be taught minutes until pain subsides, so answers A and B
not to carry the walker because this would not are incorrect. If the pain does not subside, the
provide stability; thus, answer D is incorrect. client should report to the emergency room. The
25. Answer: C. Elevate the clients hips. medication should be taken sublingually and
The client with a prolapsed cord should be should not be crushed, as stated in answer D.
treated by elevating the hips and covering the 5. Answer: C. Turkey breast
cord with a moist, sterile saline gauze. The Turkey contains the least amount of fats and
nurse should use her fingers to push up on the cholesterol. Liver, eggs, beef, cream sauces,
presenting part until a cesarean section can be shrimp, cheese, and chocolate should be
performed. Answers A, B, and D are incorrect. avoided by the client; thus, answers A, B, and D
The nurse should not attempt to replace the are incorrect. The client should bake meat rather
cord, turn the client on the side, or cover with a than frying to avoid adding fat to the meat during
dry gauze. cooking.
6. Answer: B. Neck
NCLEX Questions 8 The jugular veins in the neck should be
Answers and Rationale assessed for distension. The other parts of the
1. Answer: A. Report muscle weakness to body will be edematous in right-sided congestive
the physician. heart failure, not left-sided; thus, answers A, C,
The client taking antilipidemics should be and D are incorrect.
encouraged to report muscle weakness because 7. Answer: A. Phlebostatic axis
this is a sign of rhabdomyolysis. The medication The phlebostatic axis is located at the fifth
takes effect within 1 month of beginning therapy, intercostals space midaxillary line and is the
so answer B is incorrect. The medication should correct placement of the manometer. The PMI or
be taken with water because fruit juice, point of maximal impulse is located at the fifth
particularly grapefruit, can decrease the intercostals space midclavicular line, so answer
effectiveness, making answer C incorrect. Liver B is incorrect. Erbs point is the point at which
function studies should be checked before you can hear the valves close simultaneously,
beginning the medication, not after the fact, making answer C incorrect. The Tail of Spence
making answer D incorrect. (the upper outer quadrant) is the area where
2. Answer: B. Check the blood glucose level most breast cancers are located and has
nothing to do with placement of a manometer; answers A, C, and D might be important, they
thus, answer D is incorrect. are not the primary answer to the question.
8. Answer: B. Administer the medications 14. Answer: D. Reverse drug toxicity and
Zestril is an ACE inhibitor and is frequently given prevent tissue damage
with a diuretic such as Lasix for hypertension. Leucovorin is the antidote for Methotrexate and
Answers A, C, and D are incorrect because the Trimetrexate which are folic acid antagonists.
order is accurate. There is no need to question Leucovorin is a folic acid derivative. Answers A,
the order, administer the medication separately, B, and C are incorrect because Leucovorin does
or contact the pharmacy. not treat iron deficiency, increased neutrophils,
9. Answer: B. Measuring the extremity or have a synergistic effect.
The best indicator of peripheral edema is 15. Answer: A. HibTITER
measuring the extremity. A paper tape measure The Haemophilus influenza vaccine is given at 4
should be used rather than one of plastic or months with the polio vaccine. Answers B, C,
cloth, and the area should be marked with a and D are incorrect because these vaccines are
pen, providing the most objective assessment. given later in life.
Answer A is incorrect because weighing the 16. Answer: A. 30 minutes before meals.
client will not indicate peripheral edema. Answer Proton pump inhibitors reduce the production of
C is incorrect because checking the intake and acid in the stomach. Proton pump inhibitors work
output will not indicate peripheral edema. best when they are taken 30 minutes before the
Answer D is incorrect because checking for first meal of the day.
pitting edema is less reliable than measuring 17. Answer: A. Call security for assistance
with a paper tape measure. and prepare to sedate the client.
10. Answer: D. Visitation is limited to 30 If the client is a threat to the staff and to other
minutes when the implant is in place. clients the nurse should call for help and prepare
Clients with radium implants should have close to administer a medication such as Haldol to
contact limited to 30 minutes per visit. The sedate him. Answer B is incorrect because
general rule is limiting time spent exposed to simply telling the client to calm down will not
radium, putting distance between people and the work. Answer C is incorrect because telling the
radium source, and using lead to shield against client that if he continues he will be punished is
the radium. Teaching the family member these a threat and may further anger him. Answer D is
principles is extremely important. Answers A, B, incorrect because if the client is left alone he
and C are not empathetic and do not address might harm himself.
the question; therefore, they are incorrect. 18. Answer: A. Check the client for bladder
11. Answer: B. Split pea soup, mashed distention
potatoes, pudding, milk If the fundus of the client is displaced to the side,
The client with a facial stroke will have difficulty this might indicate a full bladder. The next action
swallowing and chewing, and the foods in by the nurse should be to check for bladder
answer B provide the least amount of chewing. distention and catheterize, if necessary. The
The foods in answers A, C, and D would require answers in B, C, and D are actions that relate to
more chewing and, thus, are incorrect. postpartal hemorrhage.
12. Answer: A. I will make sure I eat 19. Answer: C. Tuberculosis
breakfast within 10 minutes of taking my A low-grade temperature, blood-tinged sputum,
insulin. fatigue, and night sweats are symptoms
NovoLog insulin onsets very quickly, so food consistent with tuberculosis. If the answer in A
should be available within 1015 minutes of had said pneumocystis pneumonia, answer A
taking the insulin. Answer B does not address a would have been consistent with the symptoms
particular type of insulin, so it is incorrect. NPH given in the stem, but just saying pneumonia
insulin peaks in 812 hours, so a snack should isnt specific enough to diagnose the problem.
be eaten at the expected peak time. It may not Answers B and D are not directly related to the
be 3 p.m. as stated in answer C. Answer D is stem.
incorrect because there is no need to save the 20. Answer: B. Prinzmetals angina
dessert until bedtime. If the client has a history of Prinzmetals angina,
13. Answer: B. The umbilical cord needs he should not be prescribed triptan preparations
time to separate. because they cause vasoconstriction and
The umbilical cord needs time to dry and fall off coronary spasms. There is no contraindication
before putting the infant in the tub. Although for taking triptan drugs in clients with diabetes,
cancer, or cluster headaches making answers A, perform a C-section, making answer C incorrect.
C, and D incorrect. It is not enough to continue primary care, so
21. Answer: A. Pain on flexion of the hip and answer D is incorrect.
knee 27. Answer: B. Cervical cancer
Kernigs sign is positive if pain occurs on flexion The client with HPV is at higher risk for cervical
of the hip and knee. The Brudzinski reflex is and vaginal cancer related to this STI. She is not
positive if pain occurs on flexion of the head and at higher risk for the other cancers mentioned in
neck onto the chest so answer B is incorrect. answers A, C, and D, so those are incorrect.
Answers C and D might be present but are not 28. Answer: B. Herpes
related to Kernigs sign. A lesion that is painful is most likely a herpetic
22. Answer: B. Apraxia lesion. A chancre lesion associated with syphilis
Apraxia is the inability to use objects is not painful, so answer A is incorrect.
appropriately. Agnosia is loss of sensory Condylomata lesions are painless warts, so
comprehension, anomia is the inability to find answer D is incorrect. In answer C, gonorrhea
words, and aphasia is the inability to speak or does not present as a lesion, but is exhibited by
understand so answers A, C, and D are a yellow discharge.
incorrect. 29. Answer: C. Fluorescent treponemal
23. Answer: C. Sundowning antibody (FTA)
Increased confusion at night is known as Fluorescent treponemal antibody (FTA) is the
sundowning syndrome. This increased test for treponema pallidum. VDRL and RPR are
confusion occurs when the sun begins to set screening tests done for syphilis, so answers A
and continues during the night. Answer A is and B are incorrect. The Thayer-Martin culture is
incorrect because fatigue is not necessarily done for gonorrhea, so answer D is incorrect.
present. Increased confusion at night is not part 30. Answer: D. Elevated hepatic enzymes
of normal aging; therefore, answer B is incorrect. The criteria for HELLP is hemolysis, elevated
A delusion is a firm, fixed belief; therefore, liver enzymes, and low platelet count. In answer
answer D is incorrect. A, an elevated blood glucose level is not
24. Answer: C. Ill get you some juice and associated with HELLP. Platelets are
toast. Would you like something else? decreased, not elevated, in HELLP syndrome as
The client who is confused might forget that he stated in answer B. The creatinine levels are
ate earlier. Dont argue with the client. Simply elevated in renal disease and are not associated
get him something to eat that will satisfy him with HELLP syndrome so answer C is incorrect.
until lunch. Answers A and D are incorrect 31. Answer: A. The nurse places her thumb
because the nurse is dismissing the client. on the muscle inset in the antecubital space
Answer B is validating the delusion. and taps the thumb briskly with the reflex
25. Answer: D. Nausea hammer.
Nausea and gastrointestinal upset are very Answer B elicits the triceps reflex, so it is
common in clients taking acetylcholinesterase incorrect. Answer C elicits the patella reflex,
inhibitors such as Exelon. Other side effects making it incorrect. Answer D elicits the radial
include liver toxicity, dizziness, unsteadiness, nerve, so it is incorrect.
and clumsiness. The client might already be 32. Answer: B. Brethine 10 mcg IV
experiencing urinary incontinence or headaches, Brethine is used cautiously because it raises the
but they are not necessarily associated; and the blood glucose levels. Answers A, C, and D are
client with Alzheimers disease is already all medications that are commonly used in the
confused. Therefore, answers A, B, and C are diabetic client, so they are incorrect.
incorrect. 33. Answer: C. The infant is at high risk for
26. Answer: B. Report the finding to the respiratory distress syndrome.
doctor When the L/S ratio reaches 2:1, the lungs are
Any lesion should be reported to the doctor. This considered to be mature. The infant will most
can indicate a herpes lesion. Clients with open likely be small for gestational age and will not be
lesions related to herpes are delivered by at risk for birth trauma, so answer D is incorrect.
Cesarean section because there is a possibility The L/S ratio does not indicate congenital
of transmission of the infection to the fetus with anomalies, as stated in answer A, and the infant
direct contact to lesions. It is not enough to is not at risk for intrauterine growth retardation,
document the finding, so answer A is incorrect. making answer B incorrect.
The physician must make the decision to 34. Answer: C. Jitteriness
Jitteriness is a sign of seizure in the neonate. in warm weather, can exacerbate the condition.
Crying, wakefulness, and yawning are expected Answers B, C, and D are not factors for concern.
in the newborn, so answers A, B, and D are 41. Answer: D. Tell the family members to
incorrect. take the fruit home
35. Answer: B. Hypersomnolence The client with neutropenia should not have
The client is expected to become sleepy, have fresh fruit because it should be peeled and/or
hot flashes, and be lethargic. A decreasing cooked before eating. He should also not eat
urinary output, absence of the knee-jerk reflex, foods grown on or in the ground or eat from the
and decreased respirations indicate toxicity, so salad bar. The nurse should remove potted or
answers A, C, and D are incorrect. cut flowers from the room as well. Any source of
36. Answer: D. Increase the rate of the IV bacteria should be eliminated, if possible.
infusion Answers A, B, and C will not help prevent
If the client experiences hypotension after an bacterial invasions.
injection of epidural anesthetic, the nurse should 42. Answer: B. Increase the infusion of
turn her to the left side, apply oxygen by mask, Dextrose in normal saline
and speed the IV infusion. If the blood pressure In clients who have not had surgery to the face
does not return to normal, the physician should or neck, the answer would be answer A;
be contacted. Epinephrine should be kept for however, in this situation, this could further
emergency administration. Answer A is incorrect interfere with the airway. Increasing the infusion
because placing the client in Trendelenburg and placing the client in supine position would
position (head down) will allow the anesthesia to be better. Answers C is incorrect because it is
move up above the respiratory center, thereby not necessary at this time and could cause
decreasing the diaphragms ability to move up hyponatremia and further hypotension. Answer
and down and ventilate the client. In answer B, D is not necessary at this time.
the IV rate should be increased, not decreased. 43. Answer: C. Cover the insertion site with
In answer C, the oxygen should be applied by a Vaseline gauze
mask, not cannula. If the client pulls the chest tube out of the chest,
37. Answer: A. Alteration in nutrition the nurses first action should be to cover the
Cancer of the pancreas frequently leads to insertion site with an occlusive dressing.
severe nausea and vomiting and altered Afterward, the nurse should call the doctor, who
nutrition. The other problems are of lesser will order a chest x-ray and possibly reinsert the
concern; thus, answers B, C, and D are tube. Answers A, B, and D are not the first
incorrect. action to be taken.
38. Answer: C. Daily measurement of 44. Answer: A. Assess for signs of abnormal
abdominal girth bleeding
Measuring with a paper tape measure and The normal Protime is 1220 seconds. A
marking the area that is measured is the most Protime of 120 seconds indicates an extremely
objective method of estimating ascites. prolonged Protime and can result in a
Inspecting and checking for fluid waves are spontaneous bleeding episode. Answers B, C,
more subjective, so answers A and B are and D may be needed at a later time but are not
incorrect. Palpation of the liver will not tell the the most important actions to take first.
amount of ascites; thus, answer D is incorrect. 45. Answer: C. A cup of yogurt
39. Answer: B. Fluid volume deficit The food with the most calcium is the yogurt.
The vital signs indicate hypovolemic shock. Answers A, B, and D are good choices, but not
They do not indicate cerebral tissue perfusion, as good as the yogurt, which has approximately
airway clearance, or sensory perception 400 mg of calcium.
alterations, so answers A, C, and D are 46. Answer: C. The nurse inserts a Foley
incorrect. catheter.
40. Answer: A. Likes to play football The client receiving magnesium sulfate should
The client with osteogenesis imperfecta is at risk have a Foley catheter in place, and hourly intake
for pathological fractures and is likely to and output should be checked. There is no need
experience these fractures if he participates in to refrain from checking the blood pressure in
contact sports. The client might experience the right arm. A padded tongue blade should be
symptoms of hypoxia if he becomes dehydrated kept in the room at the bedside, just in case of a
or deoxygenated; extreme exercise, especially seizure, but this is not related to the magnesium
sulfate infusion. Darkening the room is
unnecessary, so answers A, B, and D are be located in the right lower quadrant, so answer
incorrect. B is incorrect. If the fetus is in the sacral
47. Answer: D. Notify the physician of the position, the FHTs will be located in the center of
mothers refusal the abdomen, so answer C is incorrect. If the
If the clients mother refuses the blood FHTs are heard in the left lower abdomen, the
transfusion, the doctor should be notified. infant is most likely in the left occiput transverse
Because the client is a minor, the court might position, making answer D incorrect.
order treatment. Answer A is incorrect. Because 3. Answer: D. Spasm of bronchial smooth
it is not the primary responsibility for the nurse to muscle
encourage the mother to consent or explain the Asthma is the presence of bronchial spasms.
consequences, so answers B and C are This spasm can be brought on by allergies or
incorrect. anxiety. Answer A is incorrect because the
48. Answer: B. Laryngeal edema primary physiological alteration is not
The nurse should be most concerned with inflammation. Answer B is incorrect because
laryngeal edema because of the area of burn. there is the production of abnormally viscous
The next priority should be answer A, as well as mucus, not a primary alteration. Answer C is
hyponatremia and hypokalemia in C and D, but incorrect because infection is not primary to
these answers are not of primary concern so are asthma.
incorrect. 4. Answer: A. Serve high-calorie foods she
49. Answer: D. The client gains weight. can carry with her
The client with anorexia shows the most The client with mania is seldom sitting long
improvement by weight gain. Selecting a enough to eat and burns many calories for
balanced diet does little good if the client will not energy. Answer B is incorrect because the client
eat, so answer A is incorrect. The hematocrit should be treated the same as other clients.
might improve by several means, such as blood Small meals are not a correct option for this
transfusion, but that does not indicate client. Allowing her into the kitchen gives her
improvement in the anorexic condition; privileges that other clients do not have and
therefore, answer B is incorrect. The tissue should not be allowed, so answer D is incorrect.
turgor indicates fluid stasis, not improvement of 5. Answer: B. Hips are slightly elevated
anorexia, so answer C is incorrect. above the bed and the legs are suspended at
50. Answer: D. Paresthesia of the toes a right angle to the bed
At this time, pain beneath the cast is normal. Bryants traction is used for fractured femurs and
The clients toes should be warm to the touch, dislocated hips. The hips should be elevated 15
and pulses should be present. Paresthesia is not off the bed. Answer A is incorrect because the
normal and might indicate compartment hips should not be resting on the bed. Answer C
syndrome. Therefore, Answers A, B, and C are is incorrect because the hips should not be
incorrect. above the level of the body. Answer D is
incorrect because the hips and legs should not
NCLEX Questions 9 be flat on the bed.
Answers and Rationale 6. Answer: B. The nurse wears gloves when
1. Answer: B. Tachycardia and diarrhea providing care.
Barbiturates create a sedative effect. When the Herpes zoster is shingles. Clients with shingles
client stops taking barbiturates, he will should be placed in contact precautions.
experience tachycardia, diarrhea, and Wearing gloves during care will prevent
tachypnea. Answer A is incorrect even though transmission of the virus. Covering the lesions
depression and suicidal ideation go along with with a sterile gauze is not necessary, antibiotics
barbiturate use; it is not the priority. Muscle are not prescribed for herpes zoster, and oxygen
cramps and abdominal pain are vague is not necessary for shingles; therefore, answers
symptoms that could be associated with other A, C, and D are incorrect.
problems. Tachycardia is associated with 7. Answer: B. 30 minutes before the infusion
stopping barbiturates, but euphoria is not. A trough level should be drawn 30 minutes
2. Answer: A. Right breech presentation before the third or fourth dose. The times in
If the fetal heart tones are heard in the right answers A, C, and D are incorrect times to draw
upper abdomen, the infant is in a breech blood levels.
presentation. If the infant is positioned in the 8. Answer: B. Keep the diaphragm in a cool
right occiput anterior presentation, the FHTs will location
The client using a diaphragm should keep the first trimester of pregnancy, as organogenesis
diaphragm in a cool location. Answers A, C, and takes place during this period. The concerns in
D are incorrect. She should refrain from leaving pregnancy are the same as for MRI in general,
the diaphragm in longer than 8 hours, not 4 but the fetus may be more sensitive to the
hours. She should have the diaphragm resized effectsparticularly to heating and to
when she gains or loses 10 pounds or has noise. Clients with a titanium hip replacement
abdominal surgery. can have an MRI. No antibiotics are used with
9. Answer: C. Im drinking four glasses of this test and the client should remain still only
fluid during a 24-hour period. when instructed, so answers C and D are not
Mothers who plan to breastfeed should drink specific to this test.
plenty of liquids, and four glasses is not enough 15. Answer: D. Changes in skin color
in a 24-hour period. Wearing a support bra is a Clients taking Amphotericin B should be
good practice for the mother who is monitored for liver, renal, and bone marrow
breastfeeding as well as the mother who plans function because this drug is toxic to the kidneys
to bottle-feed, so answer A is incorrect. and liver, and causes bone marrow suppression.
Expressing milk from the breast will stimulate Jaundice is a sign of liver toxicity and is not
milk production, making answer B incorrect. specific to the use of Amphotericin B. Changes
Allowing the water to run over the breast will in vision are not related, and nausea is a side
also facilitate letdown, when the milk begins to effect, not a sign of toxicity; nor is urinary
be produced; thus, answer D is incorrect. frequency. Thus, answers A, B, and C are
10. Answer: A. Facial pain incorrect.
The facial nerve is cranial nerve VII. If damage 16. Answer: C. The client with chest pain and
occurs, the client will experience facial pain. The a history of angina
auditory nerve is responsible for hearing loss The client with chest pain should be seen first
and tinnitus, eye movement is controlled by the because this could indicate a myocardial
Trochlear or C IV, and the olfactory nerve infarction. The client in answer A has a blood
controls smell; therefore, answers B, C, and D glucose within normal limits. The client in
are incorrect. answer B is maintained on blood pressure
11. Answer: B. Change the color of her urine medication. The client in answer D is in no
Clients taking Pyridium should be taught that the distress.
medication will turn the urine orange or red. It is 17. Answer: B. Three times per day with
not associated with diarrhea, mental confusion, meals
or changes in taste; therefore, answers A, C, Pancreatic enzymes should be given with meals
and D are incorrect. Pyridium can also cause a for optimal effects. These enzymes assist the
yellowish color to skin and sclera if taken in body in digesting needed nutrients. Answers A,
large doses. C, and D are incorrect methods of administering
12. Answer: B. Perform a pregnancy test pancreatic enzymes.
Accutane is contraindicated for use by pregnant 18. Answer: C. The lens focuses light rays on
clients because it causes teratogenic effects. the retina.
Calcium levels, apical pulse, and creatinine The lens allows light to pass through the pupil
levels are not necessary; therefore, answers A, and focus light on the retina. The lens does not
C, and D are incorrect. stimulate the retina, assist with eye movement,
13. Answer: D. Encourage fluids or magnify small objects, so answers A, B, and
Clients taking Acyclovir should be encouraged to D are incorrect.
drink plenty of fluids because renal impairment 19. Answer: C. Constrict the pupils
can occur. Limiting activity is not necessary, nor Miotic eye drops constrict the pupil and allow
is eating a high-carbohydrate diet. Use of an aqueous humor to drain out of the Canal of
incentive spirometer is not specific to clients Schlemm. They do not anesthetize the cornea,
taking Acyclovir; therefore, answers A, B, and C dilate the pupil, or paralyze the muscles of the
are incorrect. eye, making answers A, B, and D incorrect.
14. Answer: A. Pregnancy 20. Answer: A. Allow 5 minutes between the
Although there are no evidence to suggest MRI two medications.
scans can pose a risk during pregnancy, it is When using eyedrops, allow 5 minutes between
considered precaution to not perform MRI during the two medications; therefore, answer B is
pregnancy, particularly in the first three incorrect. These medications can be used by the
months. This is particularly the case during the same client but it is not necessary to use a
cycloplegic with these medications, making Success with breastfeeding depends on many
answers C and D incorrect. factors, but the most dependable reason for
21. Answer: B. Violet success is desire and willingness to continue the
Clients with color blindness will most likely have breastfeeding until the infant and mother have
problems distinguishing violets, blues, and time to adapt. The educational level, the infants
green. The colors in answers A, C, and D are birth weight, and the size of the mothers breast
less commonly affected. have nothing to do with success, so answers A,
22. Answer: D. Monitor his pulse rate B, and C are incorrect.
The client with a pacemaker should be taught to 28. Answer: C. The presence of green-tinged
count and record his pulse rate. Answers A, B, amniotic fluid
and C are incorrect. Ankle edema is a sign of Green-tinged amniotic fluid is indicative of
right-sided congestive heart failure. Although meconium staining. This finding indicates fetal
this is not normal, it is often present in clients distress. The presence of scant bloody
with heart disease. If the edema is present in the discharge is normal, as are frequent urination
hands and face, it should be reported. Checking and moderate uterine contractions, making
the blood pressure daily is not necessary for answers A, B, and D incorrect.
these clients. The client with a pacemaker can 29. Answer: C. Duration is measured by
use a microwave oven, but he should stand timing from the beginning of one contraction
about 5 feet from the oven while it is operating. to the end of the same contraction.
23. Answer: A. 1900 Duration is measured from the beginning of one
Clients who are being retrained for bladder contraction to the end of the same contraction.
control should be taught to withhold fluids after Answer A refers to frequency. Answer B is
about 7 p.m., or 1 The times in answers B, C, incorrect because we do not measure from the
and D are too early in the day. end of one contraction to the beginning of the
24. Answer: D. Drink a glass of cranberry next contraction. Duration is not measured from
juice every day. the peak of the contraction to the end, as stated
Cranberry juice is more alkaline and, when in D.
metabolized by the body, is excreted with acidic 30. Answer: B. Fetal bradycardia
urine. Bacteria does not grow freely in acidic The client receiving Pitocin should be monitored
urine. Increasing intake of meats is not for decelerations. There is no association with
associated with urinary tract infections, so Pitocin use and hypoglycemia, maternal
answer A is incorrect. The client does not have hyperreflexia, or fetal movement; therefore,
to avoid citrus fruits and peri care should be answers A, C, and D are incorrect.
done, but hydrogen peroxide is drying, so 31. Answer: D. Fetal development depends
answers B and C are incorrect. on adequate insulin regulation.
25. Answer: C. I will eat a snack around Fetal development depends on adequate
three oclock each afternoon. nutrition and insulin regulation. Insulin needs
NPH insulin peaks in 812 hours, so a snack increase during the second and third trimesters,
should be offered at that time. NPH insulin insulin requirements do not moderate as the
onsets in 90120 minutes, so answer A is pregnancy progresses, and elevated human
incorrect. Answer B is untrue because NPH chorionic gonadotrophin elevates insulin needs,
insulin is time released and does not usually not decreases them; therefore, answers A, B,
cause sudden hypoglycemia. Answer D is and C are incorrect.
incorrect, but the client should eat a bedtime 32. Answer: A. Providing a calm environment
snack. A calm environment is needed to prevent
26. Answer: B. Chest tubes serve as a seizure activity. Any stimulation can precipitate
method of draining blood and serous fluid seizures. Obtaining a diet history should be
and assist in reinflating the lungs. done later, and administering an analgesic is not
Chest tubes work to reinflate the lung and drain indicated because there is no data in the stem to
serous fluid. The tube does not equalize indicate pain. Therefore, answers B and C are
expansion of the lungs. Pain is associated with incorrect. Assessing the fetal heart tones is
collapse of the lung, and insertion of chest tubes important, but this is not the highest priority in
is painful, so answers A and C are incorrect. this situation as stated in answer D.
Answer D is true, but this is not the primary 33. Answer: A. Down syndrome
rationale for performing chest tube insertion. The client who is age 42 is at risk for fetal
27. Answer: D. Mothers desire to breastfeed anomalies such as Down syndrome and other
chromosomal aberrations. Answers B, C, and D During pregnancy, the thyroid gland triples in
are incorrect because the client is not at higher size. This makes it more difficult to regulate
risk for respiratory distress syndrome or thyroid medication. Answer A is incorrect
pathological jaundice, and Turners syndrome is because there could be a need for thyroid
a genetic disorder. medication during pregnancy. Answer C is
34. Answer: C. Dinoprostone (Prostin E.) incorrect because the thyroid function does not
The client with a missed abortion will have slow. Fetal growth is not arrested if thyroid
induction of labor. Prostin E. is a form of medication is continued, so answer D is
prostaglandin used to soften the cervix. incorrect.
Magnesium sulfate is used for preterm labor and 39. Answer: C. Cyanosis of the feet and
preeclampsia, calcium gluconate is the antidote hands
for magnesium sulfate, and Pardel is a Cyanosis of the feet and hands is acrocyanosis.
dopamine receptor stimulant used to treat This is a normal finding 1 minute after birth. An
Parkinsons disease; therefore, answers A, B, apical pulse should be 120160, and the baby
and D are incorrect. Pardel was used at one should have muscle tone, making answers A
time to dry breast milk. and B incorrect. Jaundice immediately after birth
35. Answer: A. Continue the infusion of is pathological jaundice and is abnormal, so
magnesium sulfate while monitoring the answer D is incorrect.
clients blood pressure 40. Answer: A. Supplemental oxygen
The clients blood pressure and urinary output Clients with sickle cell crises are treated with
are within normal limits. The only alteration from heat, hydration, oxygen, and pain relief. Fluids
normal is the decreased deep tendon reflexes. are increased, not decreased. Blood
The nurse should continue to monitor the blood transfusions are usually not required, and the
pressure and check the magnesium level. The client can be delivered vaginally; thus, answers
therapeutic level is 4.89.6mg/dL. Answers B, C, B, C, and D are incorrect.
and D are incorrect. There is no need to stop the 41. Answer: A. Increasing fluid intake
infusion at this time or slow the rate. Calcium Before ultrasonography, the client should be
gluconate is the antidote for magnesium sulfate, taught to drink plenty of fluids and not void. The
but there is no data to indicate toxicity. client may ambulate, an enema is not needed,
36. Answer: C. Affected parents have a one and there is no need to withhold food for 8
in four chance of passing on the defective hours. Therefore, answers B, C, and D are
gene. incorrect.
Autosomal recessive disorders can be passed 42. Answer: D. 24 pounds
from the parents to the infant. If both parents By 1 year of age, the infant is expected to triple
pass the trait, the child will get two abnormal his birth weight. Answers A, B, and C are
genes and the disease results. Parents can also incorrect because they are too low.
pass the trait to the infant. Answer A is incorrect 43. Answer: B. Measures the activity of the
because, to have an affected newborn, the fetus
parents must be carriers. Answer B is incorrect A nonstress test is done to evaluate periodic
because both parents must be carriers. Answer movement of the fetus. It is not done to evaluate
D is incorrect because the parents might have lung maturity as in answer A. An oxytocin
affected children. challenge test shows the effect of contractions
37. Answer: D. To detect neurological on fetal heart rate and a nonstress test does not
defects measure neurological well-being of the fetus, so
Alpha fetoprotein is a screening test done to answers C and D are incorrect.
detect neural tube defects such as spina bifida. 44. Answer: D. The urethral meatus opens on
The test is not mandatory, as stated in answer the underside of the penis.
A. It does not indicate cardiovascular defects, Hypospadias is a congenital abnormality in
and the mothers age has no bearing on the which the urethral meatus is on the underside of
need for the test, so answers B and C are the penis.
incorrect. 45. Answer: A. Alteration in coping related to
38. Answer: B. Regulation of thyroid pain
medication is more difficult because the Transition is the time during labor when the
thyroid gland increases in size during client loses concentration due to intense
pregnancy. contractions. Potential for injury related to
precipitate delivery has nothing to do with the
dilation of the cervix, so answer B is incorrect. compromise and provides a baseline by which to
There is no data to indicate that the client has plan further assessment and treatment. A
had anesthesia or fluid volume deficit, making thorough medical history, including onset of
answers C and D incorrect. symptoms, will be necessary and it is likely that
46. Answer: C. Antivirals an electrocardiogram will be performed as well,
Varicella is chicken pox. This herpes virus is but these are not the first priority. Similarly,
treated with antiviral medications. The client is chest exam with auscultation may offer useful
not treated with antibiotics or anticoagulants as information after vital signs are assessed.
stated in answers A and D. The client might 2. Answer: C. The patient may discontinue
have a fever before the rash appears, but when the prescribed course of oral antibiotics
the rash appears, the temperature is usually once the symptoms have completely
gone, so answer B is incorrect. resolved.
47. Answer: B. Ampicillin It is always critical that patients being
Clients with chest pain can be treated with discharged from the hospital take prescribed
nitroglycerin, a beta blocker such as propranolol, medications as instructed. In the case of
or Verapamil. There is no indication for an antibiotics, a full course must be completed even
antibiotic such as Ampicillin, so answers A, C, after symptoms have resolved to prevent
and D are incorrect. incomplete eradication of the organism and
48. Answer: B. Take prescribed anti- recurrence of infection. The patient should
inflammatory medications with meals. resume normal activities as tolerated, as well as
Anti-inflammatory drugs should be taken with a nutritious diet. Continued use of the incentive
meals to avoid stomach upset. Answers A, C, spirometer after discharge will speed recovery
and D are incorrect. Clients with rheumatoid and improve lung function.
arthritis should exercise, but not to the point of 3. Answer: C. If possible, keep the other bed
pain. Alternating hot and cold is not necessary, in the room unassigned to provide privacy
especially because warm, moist soaks are more and comfort to the family.
useful in decreasing pain. Weight-bearing When a family member is dying, it is most
activities such as walking are useful but is not helpful for nursing staff to provide a culturally
the best answer for the stem. sensitive environment to the degree possible
49. Answer: D. Morphine 8 mg IM q 4 hours within the hospital routine. In the Vietnamese
PRN pain culture, it is important that the dying be
Morphine is contraindicated in clients with surrounded by loved ones and not left alone.
gallbladder disease and pancreatitis because Traditional rituals and foods are thought to ease
morphine causes spasms of the Sphincter of the transition to the next life. When possible,
Oddi. Meperidine, Mylanta, and Cimetidine are allowing the family privacy for this traditional
ordered for pancreatitis, making answers A, B, behavior is best for them and the patient.
and C incorrect. Answers A, B, and D are incorrect because they
50. Answer: B. Hallucinogenic drugs induce create unnecessary conflict with the patient and
a state of altered perception. family.
Hallucinogenic drugs can cause hallucinations. 4. Answer: A. A one-week postoperative
Continuous observation is ordered to prevent coronary bypass patient, who is being
the client from harming himself during evaluated for placement of a pacemaker prior
withdrawal. Answers A, C, and D are incorrect to discharge.
because hallucinogenic drugs dont create both The charge nurse planning assignments must
stimulant and depressant effects or produce consider the skills of the staff and the needs of
severe respiratory depression. However, they do the patients. The labor and delivery nurse who is
produce psychological dependence rather than not experienced with the needs of cardiac
physical dependence. patients should be assigned to those with the
least acute needs. The patient who is one-week
NCLEX Questions 10 post-operative and nearing discharge is likely to
Answers and Rationale require routine care. A new patient admitted with
1. Answer: C. Careful assessment of vital suspected MI and scheduled for angiography
signs. would require continuous assessment as well as
The priority nursing action for a patient arriving coordination of care that is best carried out by
at the ED in distress is always assessment of experienced staff. The unstable patient requires
vital signs. This indicates the extent of physical staff that can immediately identify symptoms and
respond appropriately. A postoperative patient patient with significant ventricular symptoms,
also requires close monitoring and cardiac such as tachycardia resulting in syncope. A
experience. patient with myocardial infarction that resolved
5. Answer: B. Glucagon treats hypoglycemia with no permanent cardiac damage would not be
resulting from insulin overdose. a candidate. A patient recovering well from
Glucagon is given to treat insulin overdose in an coronary bypass would not need the device.
unresponsive patient. Following Glucagon Atrial tachycardia is less serious and is treated
administration, the patient should respond within conservatively with medication and
15-20 minutes at which time oral carbohydrates cardioversion as a last resort.
should be given. Glucagon reverses rather than 12. Answer: B. The patient has a pacemaker.
enhances or prolongs the effects of insulin. The implanted pacemaker will interfere with the
Lipoatrophy refers to the effect of repeated magnetic fields of the MRI scanner and may be
insulin injections on subcutaneous fat. deactivated by them. Shellfish/iodine allergy is
6. Answer: A. Sudden weight gain. not a contraindication because the contrast used
Weight gain is an early symptom of congestive in MRI scanning is not iodine-based. Open MRI
heart failure due to accumulation of fluid. When scanners and anti-anxiety medications are
diuretic therapy is inadequate, one would expect available for patients with claustrophobia.
an increase in blood pressure, tachypnea, and Psychiatric medication is not a contraindication
tachycardia to result. to MRI scanning.
7. Answer: B. 4 mcg/mL. 13. Answer: B. The patient suddenly
The therapeutic serum level for Dilantin is 10 complains of chest pain and shortness of
20 mcg/mL. A level of 4 mcg/mL is breath.
subtherapeutic and may be caused by patient Typical symptoms of pulmonary embolism
non-compliance or increased metabolism of the include chest pain, shortness of breath, and
drug. A level of 15 mcg/mL is therapeutic. severe anxiety. The physician should be notified
Choices C and D are expressed in mcg/dL, immediately. A patient with pulmonary embolism
which is the incorrect unit of measurement. will not be sleepy or have a cough with crackles
8. Answer: D. Hepatic damage. on exam. A patient with fever, chills and loss of
Acetaminophen in even moderately large doses appetite may be developing pneumonia.
can cause serious liver damage that may result 14. Answer: C. The patient will be admitted to
in death. Immediate evaluation of liver function the surgical unit and resection will be
is indicated with consideration of N- scheduled.
acetylcysteine administration as an antidote. A rapidly enlarging abdominal aortic aneurysm is
Tinnitus is associated with aspirin overdose, not at significant risk of rupture and should be
acetaminophen. Diarrhea and hypertension are resected as soon as possible. No other
not associated with acetaminophen. appropriate treatment options currently exist.
9. Answer: B. Monitor respiratory rate. 15. Answer: D. Check for signs of bleeding,
Morphine sulfate can suppress respiration and including examination of urine and stool for
respiratory reflexes, such as cough. Patients blood.
should be monitored regularly for these effects A platelet count of 25,000/microliter is severely
to avoid respiratory compromise. Morphine thrombocytopenic and should prompt the
sulfate does not significantly affect urine output, initiation of bleeding precautions, including
heart rate, or body temperature. monitoring urine and stool for evidence of
10. Answer: C. X-ray the leg. bleeding. Monitoring for fever and requiring
Following triage, an x-ray should be performed protective clothing are indicated to prevent
to rule out fracture. Ice, not heat, should be infection if white blood cells are decreased.
applied to a recent sports injury. An elastic Transfusion of red cells is indicated for severe
bandage may be applied and pain medication anemia.
given once fracture has been excluded. 16. Answer: C. We will bring in fresh flowers
11. Answer: C. A patient with a history of to brighten the room.
ventricular tachycardia and syncopal During induction chemotherapy, the leukemia
episodes. patient is severely immunocompromised and at
An automatic internal cardioverter-defibrillator risk of serious infection. Fresh flowers, fruit, and
delivers an electric shock to the heart to plants can carry microbes and should be
terminate episodes of ventricular tachycardia avoided. Books, pictures, and other personal
and ventricular fibrillation. This is necessary in a items can be cleaned with antimicrobials before
being brought into the room to minimize the risk will not speed the process and may be
of contamination. confusing.
17. Answer: A. 3-10 years. 2. Answer: C. Give only a bottle of water at
The peak incidence of ALL is at 4 years (range bedtime.
3-10). It is uncommon after the mid-teen years. Babies and toddlers should not fall asleep with
The peak incidence of chronic myelogenous bottles containing liquid other than plain water
leukemia (CML) is 45-55 years. The peak due to the risk of dental decay. Sugars in milk or
incidence of acute myelogenous leukemia (AML) juice remain in the mouth during sleep and
occurs at 60 years. Two-thirds of cases of cause caries, even in teeth that have not yet
chronic lymphocytic leukemia (CLL) occur after erupted. When water is substituted for milk or
60 years. juice, babies will often lose interest in the bottle
18. Answer: B. Night sweats and fatigue. at night.
Symptoms of Hodgkins disease include night 3. Answer: B. Allow the infant to cry for 5
sweats, fatigue, weakness, and tachycardia. The minutes before responding if she wakes
disease is characterized by painless, enlarged during the night as she may fall back asleep.
cervical lymph nodes. Weight loss occurs early Infants under 6 months may not be able to sleep
in the disease. Nausea and vomiting are not for long periods because their stomachs are too
typically symptoms of Hodgkins disease. small to hold adequate nourishment to take
19. Answer: A. Reed-Sternberg cells. them through the night. After 6 months, it may
A definitive diagnosis of Hodgkins disease is be helpful to let babies put themselves back to
made if Reed-Sternberg cells are found on sleep after waking during the night, but not prior
pathologic examination of the excised lymph to 6 months. Infants should always be placed on
node. Lymphoblasts are immature cells found in their backs to sleep. Research has shown a
the bone marrow of patients with acute dramatic decrease in sudden infant death
lymphoblastic leukemia. Gauchers cells are syndrome (SIDS) with back sleeping. Eye
large storage cells found in patients with contact and verbal engagement with infants are
Gauchers disease. Rieders cells are important to language development. The best
myeloblasts found in patients with acute diet for infants under 4 months of age is breast
myelogenous leukemia. milk or infant formula.
20. Answer: C. Stay with the patient and 4. Answers: A, B, and C
focus on slow, deep breathing for relaxation. A daily bowel movement is not necessary if the
Slow, deep breathing is the most effective patient is comfortable and the bowels move
method of reducing anxiety and stress. It regularly. Moderate exercise, such as walking,
reduces the level of carbon dioxide in the brain encourages bowel health, as does generous
to increase calm and relaxation. Warning the water intake. A diet high in fiber is also helpful.
patient to remain still will likely increase her Laxatives should be used as a last resort and
anxiety. Encouraging family members to stay should not be taken regularly. Over time,
with the patient may make her worry about their laxatives can desensitize the bowel and worsen
anxiety as well as her own. Delaying the constipation.
procedure is unlikely to allay her fears. 5. Answer: C. A negative antistreptolysin O
titer.
NCLEX Questions 11 Rheumatic fever is caused by an untreated
Answers and Rationale group A B hemolytic Streptococcus infection in
1. Answer: C. Bowel control is usually the previous 2-6 weeks, confirmed by a positive
achieved before bladder control, and the antistreptolysin O titer. Rheumatic fever is
average age for completion of toilet training characterized by a red rash over the trunk and
varies widely from 24 to 36 months. extremities as well as fever and other
Toddlers typically learn bowel control before symptoms.
bladder control, with boys often taking longer to 6. Answer: D. Check blood pressure.
complete toilet training than girls. Many children A patient with congestive heart failure and
are not trained until 36 months and this should dyspnea may have pulmonary edema, which
not cause concern. Later training is rarely can cause severe hypertension. Therefore,
caused by psychological factors and is much taking the patients blood pressure should be the
more commonly related to individual first action. Lying flat on the exam table would
developmental maturity. Reprimanding the child likely worsen the dyspnea, and the patient may
not tolerate it. Blood draws for chemistry and
ABG will be required, but not prior to the blood changes are always classified number one
pressure assessment. priority. Though the patient with chest pain
7. Answer: C. Headaches are a frequent side presented in the question recently ate a spicy
effect of nitroglycerine because it causes meal and may be suffering from heartburn, he
vasodilation. also may be having an acute myocardial
Nitroglycerin is a potent vasodilator and often infarction and require urgent attention. The
produces unwanted effects such as headache, patient with fever, headache and muscle aches
dizziness, and hypotension. Patients should be (classic flu symptoms) should be classified as
counseled, and the dose titrated, to minimize non-urgent. The patient with the foot injury may
these effects. In spite of the side effects, have sustained a sprain or fracture, and the limb
nitroglycerine is effective at reducing myocardial should be x-rayed as soon as is practical, but
oxygen consumption and increasing blood flow. the damage is unlikely to worsen if there is a
The patient should not stop the medication. delay. The childs chin laceration may need to
Nitroglycerine does not cause bleeding in the be sutured but is also non-urgent.
brain. 12. Answer: C. Hypoactive bowel sounds.
8. Answer: A. The symptoms may be the Normal serum calcium is 8.5 10 mg/dL. The
result of anemia caused by chemotherapy. patient is hypocalcemic. Increased gastric
Three months after surgery and chemotherapy motility, resulting in hyperactive (not hypoactive)
the patient is likely to be feeling the after-effects, bowel sounds, abdominal cramping and diarrhea
which often includes anemia because of bone- is an indication of hypocalcemia. Numbness in
marrow suppression. There is no evidence that hands and feet and muscle cramps are also
the patient is immunosuppressed, and fatigue is signs of hypocalcemia. Positive Chvosteks sign
not a typical symptom of immunosuppression. refers to the sustained twitching of facial
The information given does not indicate that muscles following tapping in the area of the
depression or dehydration is a cause of her cheekbone and is a hallmark of hypocalcemia.
symptoms. 13. Answer: A. pH 7.52, PCO2 54 mmHg.
9. Answer: C. The patient should use iron A patient on nasogastric suction is at risk of
cookware to prepare foods, such as dark metabolic alkalosis as a result of loss of
green, leafy vegetables and legumes, which hydrochloric acid in gastric fluid. Of the answers
are high in iron. given, only answer A (pH 7.52, PCO2 54 mm
Normal hemoglobin values range from 11.5- Hg) represents alkalosis. Answer B is a normal
15.0. This vegetarian patient is mildly anemic. blood gas. Answer C represents respiratory
When food is prepared in iron cookware its iron acidosis. Answer D is borderline normal with
content is increased. In addition, dark green slightly low PCO2.
leafy vegetables, such as spinach and kale, and 14. Answer: A. Draw a blood sample for
legumes are high in iron. Mild anemia does not prothrombin (PT) and international
require that animal sources of iron be added to normalized ratio (INR) level.
the diet. Many non-animal sources are available. The effect of Coumadin is to inhibit clotting. The
Coffee and tea increase gastrointestinal activity next step is to check the PT and INR to
and inhibit absorption of iron. determine the patients anticoagulation status
10. Answer: D. A nurse should remain in the and risk of bleeding. Vitamin K is an antidote to
room during the first 15 minutes of infusion. Coumadin and may be used in a patient who is
Transfusion reaction is most likely during the at imminent risk of dangerous bleeding.
first 15 minutes of infusion, and a nurse should Preparation for transfusion, as described in
be present during this period. PRBCs should be option C, is only indicated in the case of
infused through a 19g or larger IV catheter to significant blood loss. If lab results indicate an
avoid slow flow, which can cause clotting. anticoagulation level that would place the patient
PRBCs must be flushed with 0.45% normal at risk of excessive bleeding, the surgeon may
saline solution. Other intravenous solutions will choose to delay surgery and discontinue the
hemolyze the cells. medication.
11. Answer: C. A patient with abdominal and 15. Answer: A and B
chest pain following a large, spicy meal. Normal hemoglobin in adults is 12 16 g/dL.
Emergency triage involves quick patient Total cholesterol levels of 200 mg/dL or below
assessment to prioritize the need for further are considered normal. Total serum protein of
evaluation and care. Patients with trauma, chest 7.0-g/dL and glycosylated hemoglobin A1c of
pain, respiratory distress, or acute neurological 5.4% are both normal levels.
16. Answer: C. Nonsteroidal anti- The parathyroid glands regulate the calcium
inflammatory drugs are the first choice in level in the blood. In hyperparathyroidism, the
treatment. serum calcium level will be elevated. Parathyroid
Nonsteroidal anti-inflammatory drugs are hormone levels may be high or normal but not
important first line treatment for juvenile low. The body will lower the level of vitamin D in
idiopathic arthritis (formerly known as juvenile an attempt to lower calcium. Urine calcium may
rheumatoid arthritis). NSAIDs require 3-4 weeks be elevated, with calcium spilling over from
for the therapeutic anti-inflammatory effects to elevated serum levels. This may cause renal
be realized. Half of children with the disorder stones.
recover without joint deformity, and about a third 2. Answer: D. A restricted sodium diet.
will continue with symptoms into adulthood. A patient with Addisons disease requires normal
Physical activity is an integral part of therapy. dietary sodium to prevent excess fluid loss.
17. Answer: B. A blood culture is drawn. Adequate caloric intake is recommended with a
Antibiotics must be started after the blood diet high in protein and complex carbohydrates,
culture is drawn, as they may interfere with the including grains.
identification of the causative organism. The 3. Answer: C. Hypoglycemia.
blood count will reveal the presence of infection A postoperative diabetic patient who is unable to
but does not help identify an organism or guide eat is likely to be suffering from hypoglycemia.
antibiotic treatment. Parental presence is Confusion and shakiness are common
important for the adjustment of the child but not symptoms. An anesthesia reaction would not
for the administration of medication. occur on the second post-operative day.
18. Answer: A. Possible fracture of the tibia. Hyperglycemia and ketoacidosis do not cause
The childs refusal to walk, combined with confusion and shakiness.
swelling of the limb is suspicious for fracture. 4. Answer: A. Bowel perforation.
Toddlers will often continue to walk on a muscle Bowel perforation is the most serious
that is bruised or strained. The radius is found in complication of fiberoptic colonoscopy.
the lower arm and is not relevant to this Important signs include progressive abdominal
question. Toddlers rarely feign injury to be pain, fever, chills, and tachycardia, which
carried, and swelling indicates a physical injury. indicate advancing peritonitis. Viral
19. Answers: A, B and D. gastroenteritis and colon cancer do not cause
Delayed developmental milestones are these symptoms. Diverticulitis may cause pain,
characteristic of cerebral palsy, so regular fever, and chills, but is far less serious than
screening and intervention is essential. Because perforation and peritonitis.
of injury to upper motor neurons, children may 5. Answer: A, B, and C
have ocular and speech difficulties. Parent Prothrombin time, partial thromboplastin time,
support groups help families to share and cope. and platelet count are all included in coagulation
Physical therapy and other interventions can studies. The hemoglobin level, though important
minimize the extent of the delay in information prior to an invasive procedure like
developmental milestones. liver biopsy, does not assess coagulation.
20. Answer: A. Duchennes is an X-linked 6. Answer: D. The right side of the sternum
recessive disorder, so daughters have a 50% just below the clavicle and left of the
chance of being carriers and sons a 50% precordium.
chance of developing the disease. One gel pad should be placed to the right of the
The recessive Duchenne gene is located on one sternum, just below the clavicle and the other
of the two X chromosomes of a female carrier. If just left of the precordium, as indicated by the
her son receives the X bearing the gene he will anatomic location of the heart. To defibrillate,
be affected. Thus, there is a 50% chance of a the paddles are placed over the pads. Options
son being affected. Daughters are not affected, A, B, and C are not consistent with the position
but 50% are carriers because they inherit one of the heart and are therefore incorrect
copy of the defective gene from the mother. The responses.
other X chromosome comes from the father, 7. Answer: D. All of the above.
who cannot be a carrier. All of the statements are true. The gurgles and
clicks described in the question represent
NCLEX Questions 12 normal bowel sounds, which vary with the phase
Answers and Rationale of digestion. Intestinal obstruction causes the
1. Answer: A. Elevated serum calcium. sounds to intensify as the normal flow is blocked
by the obstruction. The swishing and buzzing 13. Answer: A. Observe for evidence of
sound of turbulent blood flow may be heard in spontaneous bleeding.
the abdomen in the presence of abdominal Platelet counts under 30,000/microliter may
aortic aneurism, for example, and should always cause spontaneous petechiae and bruising,
be considered abnormal. particularly in the extremities. When the count
8. Answer: A. Irrigate the eye repeatedly with falls below 15,000, spontaneous bleeding into
normal saline solution. the brain and internal organs may occur.
Emergency treatment following a chemical Headaches may be a sign and should be
splash to the eye includes immediate irrigation watched for. Aspirin disables platelets and
with normal saline. The irrigation should be should never be used in the presence of
continued for at least 10 minutes. Fluorescein thrombocytopenia. Thrombocytopenia does not
drops are used to check for scratches on the compromise immunity, and there is no reason to
cornea due to their fluorescent properties and limit visitors as long as any physical trauma is
are not part of the initial care of a chemical prevented.
splash, nor is patching the eye. Following 14. Answers: A, B, and D
irrigation, visual acuity will be assessed. Side effects of corticosteroids include weight
9. Answer: D. Temperature of 101.8 F (38.7 gain, fluid retention with hypertension,
C). Cushingoid features, a low serum albumin, and
Post-surgical nursing assessment after hip suppressed inflammatory response. Patients are
replacement should be principally concerned encouraged to eat a diet high in protein,
with the risk of neurovascular complications and vitamins, and minerals and low in sodium.
the development of infection. A temperature of Corticosteroids cause hypernatremia, not
101.8 F (38.7 C) postoperatively is higher than hyponatremia.
the low grade that is to be expected and should 15. Answer: B. Change gloves immediately
raise concern. Some pain during repositioning after use.
and following physical therapy is to be expected The neutropenic patient is at risk of infection.
and can be managed with analgesics. A small Changing gloves immediately after use protects
amount of bloody drainage on the surgical patients from contamination with organisms
dressing is a result of normal healing. picked up on hospital surfaces. This
10. Answer: B. Restrain the patients limbs. contamination can have serious consequences
During a witnessed seizure, nursing actions for an immunocompromised patient. Changing
should focus on securing the patients safely and the respiratory mask is desirable, but not nearly
curtailing the seizure. Restraining the limbs is as urgent as changing gloves. Minimizing
not indicated because strong muscle contact and conversation are not necessary and
contractions could cause injury. A side-lying may cause nursing staff to miss changes in the
position with head flexed forward allows for patients symptoms or condition.
drainage of secretions and prevents the tongue 16. Answer: D. Immunization provides
from falling back, blocking the airway. Rectal acquired immunity from some specific
diazepam may be a treatment ordered by the diseases.
physician, who should be notified of the seizure. Immunization is available for the prevention of
11. Answer: B. An increase in hematocrit. some, but not all, specific diseases. This type of
Epoetin is a form of erythropoietin, which immunity is acquired by causing antibodies to
stimulates the production of red blood cells, form in response to a specific pathogen. Natural
causing an increase in hematocrit. Epoetin is immunity is present at birth because the infant
given to patients who are anemic, often as a acquires maternal antibodies Immunization, like
result of chemotherapy treatment. Epoetin has all medication, cannot be risk-free and should be
no effect on neutrophils, platelets, or serum iron. considered based on the risk of the disease in
12. Answer: B, C, and D question.
Polycythemia vera is a condition in which the 17. Answer: B. Maintain a patent airway.
bone marrow produces too many red blood The patient may be experiencing an
cells. This causes an increase in hematocrit and anaphylactic reaction. The most urgent action is
viscosity of the blood. Patients can experience to maintain an airway, particularly with visible
headaches, dizziness, and visual disturbances. oral swelling, followed by the administration of
Cardiovascular effects include increased blood epinephrine by subcutaneous injection. The
pressure and delayed clotting time. Weight loss physician will see the patient as soon as
is not a manifestation of polycythemia vera. possible with the above actions underway. Oral
diphenhydramine is indicated for mild allergic days after the infection, resulting in scant, dark
reactions and is not appropriate for anaphylaxis. urine and retention of body fluid. Periorbital
18. Answers: A, B, and D edema and hypertension are common signs at
ADHD in children is frequently treated with CNS diagnosis.
stimulant medications, which increase focus and 4. Answer: C. No treatment is necessary; the
improve concentration. Children often fluid is reabsorbing normally.
experience insomnia, agitation, and decreased A hydrocele is a collection of fluid in the scrotum
appetite. Sleepiness is not a side effect of that results from a patent tunica vaginalis.
stimulants. Illumination of the scrotum with a pocket light
19. Answer: D. Tardive dyskinesia. demonstrates the clear fluid. In most cases the
Abnormal facial movements and tongue fluid reabsorbs within the first few months of life
protrusion in a patient taking haloperidol is most and no treatment is necessary. Massaging the
likely due to tardive dyskinesia, an adverse area or placing the infant in a supine position
reaction to the antipsychotic. Depression may would have no effect. Surgery is not indicated.
occur along with schizophrenia and would be 5. Answer: A. Inadequate tissue perfusion
characterized by such symptoms as loss of leading to nerve damage.
affect, appetite and/or sleep changes, and Patients with peripheral vascular disease often
anhedonia. These depressive changes and lack sustain nerve damage as a result of inadequate
of volition are part of the negative symptoms of tissue perfusion. Fluid overload is not
schizophrenia. Psychotic hallucinations may be characteristic of PVD. There is nothing to
visual or auditory but do not include abnormal indicate psychiatric disturbance in the patient.
movements. Skin changes in PVD are secondary to
20. Answer: B. Confusion. decreased tissue perfusion rather than primary
Hypoglycemia in diabetes mellitus causes inflammation.
confusion, indicating the need for carbohydrates. 6. Answer: B. Contaminated food.
Polydipsia, blurred vision, and polyphagia are Hepatitis A is the only type that is transmitted by
symptoms of hyperglycemia. the fecal-oral route through contaminated food.
Hepatitis B, C, and D are transmitted through
NCLEX Questions 13 infected bodily fluids.
Answers and Rationale 7. Answer: A. A history of hepatitis C five
1. Answer: C. The tumor extended beyond years previously.
the kidney but was completely resected. Hepatitis C is a viral infection transmitted
The staging of Wilms tumor is confirmed at through bodily fluids, such as blood, causing
surgery as follows: Stage I, the tumor is limited inflammation of the liver. Patients with hepatitis
to the kidney and completely resected; stage II, C may not donate blood for transfusion due to
the tumor extends beyond the kidney but is the high risk of infection in the recipient.
completely resected; stage III, residual non Cholecystitis (gallbladder disease),
hematogenous tumor is confined to the diverticulosis, and history of Crohns disease do
abdomen; stage IV, hematogenous metastasis not preclude blood donation.
has occurred with spread beyond the abdomen; 8. Answer: A. Naproxen sodium (Naprosyn).
and stage V, bilateral renal involvement is Naproxen sodium is a nonsteroidal anti-
present at diagnosis. inflammatory drug that can cause inflammation
2. Answers: A, B, and C of the upper GI tract. For this reason, it is
Acute glomerulonephritis is characterized by contraindicated in a patient with gastritis.
high urine specific gravity related to oliguria as Calcium carbonate is used as an antacid for the
well as dark tea colored urine caused by large relief of indigestion and is not contraindicated.
amounts of red blood cells. There is periorbital Clarithromycin is an antibacterial often used for
edema, but generalized edema is seen in the treatment of Helicobacter pylori in gastritis.
nephrotic syndrome, not acute Furosemide is a loop diuretic and is
glomerulonephritis. contraindicated in a patient with gastritis.
3. Answer: B. Prior infection with group A 9. Answer: D. The patient should limit fatty
Streptococcus within the past 10-14 days. foods.
Acute glomerulonephritis is most commonly Cholecystitis, inflammation of the gallbladder, is
caused by the immune response to a prior upper most commonly caused by the presence of
respiratory infection with group A Streptococcus. gallstones, which may block bile (necessary for
Glomerular inflammation occurs about 10-14 fat absorption) from entering the intestines.
Patients should decrease dietary fat by limiting not lie down. ACE inhibitors are not
foods like fatty meats, fried foods, and creamy contraindicated with alendronate and there is no
desserts to avoid irritation of the gallbladder. iodine allergy relationship.
10. Answer: D. Air hunger. 15. Answer: C. Prophylactic antibiotic
Patients with pulmonary edema experience air therapy prior to anticipated exposure to
hunger, anxiety, and agitation. Respiration is ticks.
fast and shallow and heart rate increases. Prophylactic use of antibiotics is not indicated to
Stridor is noisy breathing caused by laryngeal prevent Lyme disease. Antibiotics are used only
swelling or spasm and is not associated with when symptoms develop following a tick bite.
pulmonary edema. Insect repellant should be used on skin and
11. Answer: D. Change the dressing and clothing when exposure is anticipated. Clothing
document the clean appearance of the should be designed to cover as much exposed
wound site. area as possible to provide an effective barrier.
A moderate amount of serous drainage from a Close examination of skin and hair can reveal
recent surgical site is a sign of normal healing. the presence of a tick before a bite occurs.
Purulent drainage would indicate the presence 16. Answer: B. The area proximal to the
of infection. A soiled dressing should be insertion site is reddened, warm, and painful.
changed to avoid bacterial growth and to An IV site that is red, warm, painful and swollen
examine the appearance of the wound. The indicates that phlebitis has developed and the
surgical site is typically covered by gauze line should be discontinued and restarted at
dressings for a minimum of 48-72 hours to another site. Pain on movement should be
ensure that initial healing has begun. managed by maneuvers such as splinting the
12. Answer: C. Severe pain in the right lower limb with an IV board or gently shifting the
arm. position of the catheter before making a decision
Impaired perfusion to the right lower arm as a to remove the line. An IV line that is running
result of a closed cast may cause neurovascular slowly may simply need flushing or
compromise and severe pain, requiring repositioning. A hematoma at the site is likely a
immediate cast removal. Itching under the cast result of minor bleeding at the time of insertion
is common and fairly benign. Neurovascular and does not require discontinuation of the line.
compromise in the arm would not cause pain in 17. Answer: D. Fluid overload.
the shoulder, as perfusion there would not be Fluid overload occurs when then the fluid
affected. Impaired perfusion would cause the volume infused over a short period is too great
fingers to be cool and pale. Increased warmth for the vascular system, causing fluid leak into
would indicate increased blood flow or infection. the lungs. Symptoms include dyspnea, rapid
13. Answer: A. Increased physical activity respirations, and discomfort as in the patient
and daily exercise will help decrease described. Febrile non-hemolytic reaction results
discomfort associated with the condition. in fever. Symptoms of allergic transfusion
Physical activity and daily exercise can help to reaction would include flushing, itching, and a
improve movement and decrease pain in generalized rash. Acute hemolytic reaction may
osteoarthritis. Joint pain and stiffness are often occur when a patient receives blood that is
at their worst during the early morning after incompatible with his blood type. It is the most
several hours of decreased movement. serious adverse transfusion reaction and can
Acetaminophen is a pain reliever, but does not cause shock and death.
have anti-inflammatory activity. Ibuprofen is a 18. Answer: B, C, and D
strong anti-inflammatory, but should always be Uterine contractions typically become stronger
taken with food to avoid GI distress. and occur more closely together following
14. Answer: D. A patient on bed rest who amniotomy. The FHR is assessed immediately
must maintain a supine position. after the procedure and followed closely to
Alendronate can cause significant detect changes that may indicate cord
gastrointestinal side effects, such as esophageal compression. The procedure itself is painless
irritation, so it should not be taken if a patient and results in the quick expulsion of amniotic
must stay in supine position. It should be taken fluid. Following amniotomy, cervical checks are
upon rising in the morning with 8 ounces of minimized because of the risk of infection
water on an empty stomach to increase 19. Answer: D. Keep the baby quiet and
absorption. The patient should not eat or drink swaddled, and place the bassinet in a dimly
for 30 minutes after administration and should lit area.
An infant discharged home with increase physical fitness, nor is it intended to
hyperbilirubinemia (newborn jaundice) should be prevent bedsores or constipation.
placed in a sunny rather than dimly lit area with 5. Answer: D. Confusion.
skin exposed to help process the bilirubin. Cardiogenic shock severely impairs the pumping
Frequent feedings will help to metabolize the function of the heart muscle, causing diminished
bilirubin. A recheck of the serum bilirubin and a blood flow to the organs of the body. This results
physical exam within 72 hours will confirm that in diminished brain function and confusion, as
the level is falling and the infant is thriving and is well as hypotension, tachycardia, and weak
well hydrated. Signs of dehydration, including pulse. Cardiogenic shock is a serious
decreased urine output and skin changes, complication of myocardial infarction with a high
indicate inadequate fluid intake and will worsen mortality rate.
the hyperbilirubinemia. 6. Answer: A. Family history of heart disease.
20. Answer: A. The infant should be Family history of heart disease is an inherited
restrained in an infant car seat, properly risk factor that is not subject to lifestyle change.
secured in the back seat in a rear-facing Having a first degree relative with heart disease
position. has been shown to significantly increase risk.
All infants under 1 year of age weighing less Overweight and smoking are risk factors that are
than 20 lbs. should be placed in a rear-facing subject to lifestyle change and can reduce risk
infant car seat secured properly in the back seat. significantly. Advancing age increases risk of
Infant car seats should never be placed in the atherosclerosis but is not a hereditary factor.
front passenger seat. Infants should always be 7. Answers: A, C, and D
placed in an approved car seat during travel, Claudication describes the pain experienced by
even on that first ride home from the hospital. a patient with peripheral vascular disease when
oxygen demand in the leg muscles exceeds the
NCLEX Questions 14 oxygen supply. This most often occurs during
Answers and Rationale activity when demand increases in muscle
1. Answer: C. Decreased pain. tissue. The tissue becomes hypoxic, causing
Furosemide, a loop diuretic, does not alter pain. cramping, weakness, and discomfort.
Furosemide acts on the kidneys to increase 8. Answer: C. Avoid crossing the legs.
urinary output. Fluid may move from the Patients with peripheral vascular disease should
periphery, decreasing edema. Fluid load is avoid crossing the legs because this can impede
reduced, lowering blood pressure. blood flow. Walking barefoot is not advised, as
2. Answer: A. Obesity. foot protection is important to avoid trauma that
Obesity is an important risk factor for coronary may lead to serious infection. Heating pads can
artery disease that can be modified by improved cause injury, which can also increase the risk of
diet and weight loss. Family history of coronary infection. Skin lesions at risk for infection should
artery disease, male gender, and advancing age be examined and treated by a physician.
increase risk but cannot be modified. 9. Answer: C. A young woman.
3. Answer: B. History of cerebral Raynauds disease is most common in young
hemorrhage. women and is frequently associated with
A history of cerebral hemorrhage is a rheumatologic disorders, such as lupus and
contraindication to tPA because it may increase rheumatoid arthritis.
the risk of bleeding. TPA acts by dissolving the 10. Answer: B. Pulmonary embolism due to
clot blocking the coronary artery and works best deep vein thrombosis (DVT).
when administered within 6 hours of onset of In a hospitalized patient on prolonged bed rest,
symptoms. Prior MI is not a contraindication to he most likely cause of sudden onset shortness
tPA. Patients receiving tPA should be observed of breath and chest pain is pulmonary embolism.
for changes in blood pressure, as tPA may Pregnancy and prolonged inactivity both
cause hypotension. increase the risk of clot formation in the deep
4. Answer: C. Prevents DVT (deep vein veins of the legs. These clots can then break
thrombosis). loose and travel to the lungs. Myocardial
Exercise is important for all hospitalized patients infarction and atherosclerosis are unlikely in a
to prevent deep vein thrombosis. Muscular 27-year-old woman, as is congestive heart
contraction promotes venous return and failure due to fluid overload. There is no reason
prevents hemostasis in the lower extremities. to suspect an anxiety disorder in this patient.
This exercise is not sufficiently vigorous to Though anxiety is a possible cause of her
symptoms, the seriousness of pulmonary unemployment, unstable financial situation, and
embolism demands that it be considered first. single status.
11. Answer: B. Cerebral hemorrhage. 16. Answer: B. Repeated vomiting.
Cerebral hemorrhage is a significant risk when Increased pressure caused by bleeding or
treating a stroke victim with thrombolytic therapy swelling within the skull can damage delicate
intended to dissolve a suspected clot. Success brain tissue and may become life threatening.
of the treatment demands that it be instituted as Repeated vomiting can be an early sign of
soon as possible, often before the cause of pressure as the vomit center within the medulla
stroke has been determined. Air embolism is not is stimulated. The anterior fontanel is closed in a
a concern. Thrombolytic therapy does not lead 4-year-old child. Evidence of sleepiness at 10
to expansion of the clot, but to resolution, which PM is normal for a four year old. The average 4-
is the intended effect. year-old child cannot read yet, so this too is
12. Answer: A. Torticollis, with shortening of normal.
the sternocleidomastoid muscle. 17. Answer: A. Small blue-white spots are
In torticollis, the sternocleidomastoid muscle is visible on the oral mucosa.
contracted, limiting range of motion of the neck Kopliks spots are small blue-white spots visible
and causing the chin to point to the opposing on the oral mucosa and are characteristic of
side. In craniosynostosis one of the cranial measles infection. The body rash typically
sutures, often the sagittal, closes prematurely, begins on the face and travels downward. High
causing the head to grow in an abnormal shape. fever is often present. Tear drop on a rose
Plagiocephaly refers to the flattening of one side petal refers to the lesions found in varicella
of the head, caused by the infant being placed (chicken pox).
supine in the same position over time. 18. Answer: C. Petechiae occur on the soft
Hydrocephalus is caused by a build-up of palate.
cerebrospinal fluid in the brain resulting in large Petechiae on the soft palate are characteristic of
head size. rubella infection. Choices A, B, and D are
13. Answer: C. The student experiences pain characteristic of scarlet fever, a result of group A
in the inferior aspect of the knee. Streptococcus infection.
Osgood-Schlatter disease occurs in adolescents 19. Answer: B. The dose is too low.
in rapid growth phase when the infrapatellar This child weighs 30 kg, and the pediatric dose
ligament of the quadriceps muscle pulls on the of diphenhydramine is 5 mg/kg/day (5 X 30 =
tibial tubercle, causing pain and swelling in the 150/day). Therefore, the correct dose is 150
inferior aspect of the knee. Osgood-Schlatter mg/day. Divided into 3 doses per day, the child
disease is commonly caused by activities that should receive 50 mg 3 times a day rather than
require repeated use of the quadriceps, 25 mg 3 times a day. Dosage should not be
including track and soccer. Swimming is not a titrated based on symptoms without consulting a
likely cause. The condition is usually self-limited, physician.
responding to ice, rest, and analgesics. 20. Answer: D. Normally, the testes descend
Continued participation will worsen the condition by one year of age.
and the symptoms. Normally, the testes descend by one year of
14. Answer: D. Scoliosis. age. In young infants, it is common for the testes
A check for scoliosis, a lateral deviation of the to retract into the inguinal canal when the
spine, is an important part of the routine environment is cold or the cremasteric reflex is
adolescent exam. It is assessed by having the stimulated. Exam should be done in a warm
teen bend at the waist with arms dangling, while room with warm hands. It is most likely that both
observing for lateral curvature and uneven rib testes are present and will descend by a year. If
level. Scoliosis is more common in female not, a full assessment will determine the
adolescents. Choices A, B, and C are not part of appropriate treatment.
the routine adolescent exam.
15. Answer: C. Self-blame for the injury to NURSING RESEARCH 1
the child. Answers and Rationale
The profile of a parent at risk of abusive 1. Answer: C, D
behavior includes a tendency to blame the child 2. Answer: B, C
or others for the injury sustained. These parents 3. Answer: B, C
also have a high incidence of low self-esteem, 4. Answer: D. A journal article about a study
that used large, previously unpublished
databases generated by the United States NURSING RESEARCH 2
census Answers and Rationale
This is a report of an original study, so it is the 1. Answer: B. Hypotheses help frame a test
primary source of the study. of the validity of a theory.
5. Answer: C. The Cochrane Statistical Although theories cannot be tested directly,
Methods hypotheses provide a bridge between theory
6. Answer: A. Databased literature and the real world. It is the research question
Audio and video recordings of research that represents the main idea to be studied (A).
presentations are examples of databased Theories cannot be tested directly (C). The
literature. research question is also called the problem
7. Answer: C. Review of literature statement (D).
Review of literature is the 1st step in the 2. Answer: B. Meditation techniques found to
qualitative research process. Data analysis is be effective
the sixth step in the qualitative research 3. Answer: C. It is manipulated by the
process. Sampling is the third step in the researcher.
qualitative research process. The study design The independent variable is manipulated by the
is the second step in the qualitative research researcher and has a presumed effect on the
process. dependent variable. It is the dependent variable
8. Answer: B. Anticipatory guidance that is predicted to change (A). The independent
Anticipatory guidance is the sharing of variable is presumed to change the dependent
qualitative findings with the patient. Other variable (B). The independent variable is
options are not considered to be the sharing of manipulated by the researcher and is identified
qualitative findings with the patient. at the beginning of the study (D).
9. Answer: A, B, D 4. Answer: B. Hypotheses are statements
10. Answer: B, C, D about the relationships among variables.
11. Answer: A, C, E, F Hypotheses are statements about the
12. Answer: A, D relationships between two or more variables that
13. Answer: A, C suggest an answer to the research question.
14. Answer: B. What is the meaning of health Hypotheses are not concerned with
for migrant farm-worker women? operationally defining the variables involved in
This question seeks to explore a phenomenon the study (A). The independent variable is not
(health) for a specific population. affected or changed by the dependent variable
15. Answer: D. Subjects were asked to (C). Hypotheses are not concerned with
relate their perceptions of pain? operationally defining the variables involved in
Data collected were perceptions of pain, not the study, including treatments or interventions
numeric data. Other options are found in a (D).
report of a quantitative study. 5. Answer: D. It implies a causative or
16. Answer: A. A convenience sample was associative relationship.
chosen? A hypothesis implies a causative or associative
When a sample of convenience is chosen, the relationship. A hypothesis guides the research
study is a quantitative study. Qualitative studies design and collection of data (A). Operational
explore phenomena. Data collected in qualitative definitions are not included in the hypothesis (B).
studies are interpreted. Qualitative studies The hypothesis indicates the dependent variable
explore the meaning of human experience. (C).
17. Answer: B, D 6. Answer: B. After development of the
18. Answer: A, C, D research question
19. Answer: A, B The hypothesis is developed after development
Quantifiable words increase the testability of a of the research question.
hypothesis (A). The more clearly the hypothesis 7. Answer: D. The ability to meditate causes
is stated, the easier it will be to accept or reject it lower anxiety in patients with anxiety
based on study findings (B). Hypotheses should disorder than those who do not meditate.
not have value-laden words (C). Data-collection This hypothesis meets the criteria of testability.
processes are not part of the criterion used to 8. Answer: A. It is a null hypothesis.
evaluate the testability of hypotheses (D). Statistical hypotheses, called null hypotheses,
20. Answer: A, B state that there is no relationship between the
independent and dependent variables.
9. Answer: C. The independent and 1. Answer: B. To generate theory from data
dependent variables are related. The grounded theory method refers to a
Because the null hypothesis states that there is qualitative approach of building theory about a
no relationship between the independent and phenomenon about which little is known.
dependent variables, it is rejected if they are 2. Answer: B. The research method that best
related. meets intended purpose of the study should
10. Answer: C. Level III be used.
Evidence provided by quasi-experimental Different research methods accomplish different
studies is level III. Level I evidence is obtained goals and offer different types and levels of
from a systematic review of all randomized, evidence that inform practice.
controlled trials. Level II evidence is obtained 3. Answer: C. Actualization
from at least one well-designed randomized, 4. Answer: C. Are the informants who were
controlled trial. Level IV evidence is obtained chosen appropriate to inform the research?
from nonexperimental studies. 5. Answer: A. Is the strategy used for
11. Answer: D. It is a hypothesis-generating analysis compatible with the purpose of the
study. study?
Not enough is known in this area at this time to 6. Answer: B. What is the projected
formulate hypotheses, so the researcher will significance of the work to nursing?
conduct this qualitative study and use the 7. Answer: D. Has adequate time been
findings to generate hypotheses for future allowed to understand fully the
studies. This is a qualitative study, not a quasi- phenomenon?
experimental study. Level II evidence is obtained 8. Answer: B. Does the researcher document
from at least one well-designed randomized, the research process?
controlled trial. This study has no hypothesis. 9. Answer: A. Is the strategy used for
12. Answer: A. Degree of treatment received analysis compatible with the purpose of the
The degree of treatment received is considered study?
the dependent variable. 10. Answer: A, C, D
13. Answer: B. Age of the patient 11. Answer: A, B, C
The age of the patient would be the independent 12. Answer: A, D
variable. 13. Answer: D. It is an inductive approach.
14. Answer: B. Hypothesis Data are collected using the emic perspective
A hypothesis is the tool of quantitative studies, (A). The grounded-theory method is a process of
and is only found in such studies. constructing theory from human experience
15. Answer: A, B (B). In grounded-theory only primary sources
16. Answer: A, D (the participants) are used (C).
17. Answer: A, C 14. Answer: B. Constant-comparative
18. Answer: B. Constructivism method
The paradigm that provides the basis for 15. Answer: A. It yields a better
qualitative research is constructivism. understanding of each case.
19. Answer: C. Quantitative research An intrinsic case study is undertaken to have a
In qualitative research, researchers are never better understanding of the case.
considered neutral (A). In ethnography, a type of 16. Answer: B. It synthesizes critical masses
qualitative research, researchers are never of qualitative findings.
considered neutral (B). In case studies, a type of 17. Answer: C. Faithfulness to everyday
qualitative research, researchers are never reality of the participants
considered neutral (D). Credibility is the truth of findings as judged by
20. Answer: D. Quantitative the participants (A). Auditability assists the
The values of the researcher must be reader to judge the appropriateness of the
acknowledged in qualitative research (A). The interview questions posed (B). Auditability
values of the researcher must be acknowledged assists the reader to judge the adequacy of the
in naturalistic research (B). The values of the coding system used (D).
researcher must be acknowledged in qualitative 18. Answer: B, C, D
research (C). 19. Answer: B, C
20. Answer: A. Do the participants recognize
NURSING RESEARCH 3 the experience as their own?
Answers and Rationale
Prioritization, Delegation and Assignment 1 The patient with asthma did not achieve relief
Answers and Rationale from shortness of breath after using the
1. Answer: A. Perform postural drainage and bronchodilator and is at risk for respiratory
chest physiotherapy every 4 hours complications. This patients needs are urgent.
Airway clearance techniques are critical for The other patients need to be assessed as soon
patients with cystic fibrosis and should take as possible, but none of their situations are
priority over the other activities. Although urgent. in COPD patients pulse oximetry oxygen
allowing more independent decision making is saturations of more than 90% are acceptable.
important for adolescents, the physiologic need 7. Answer: D. Tympanic temperature of 101.4
for improved respiratory function takes F (38.6 C)
precedence at this time. A private room may be Infections are always a threat for the patient
desirable for the patient but is not necessary. receiving mechanical ventilation. The
With increased shortness of breath, it will be endotracheal tube bypasses the bodys normal
more important that the patient have frequent air-filtering mechanisms and provides a direct
respiratory treatments than 8 hours of sleep. access route for bacteria or viruses to the lower
2. Answers: A, B, C, and E. part of the respiratory system.
While a patient is receiving anticoagulation 8. Answers: A and B.
therapy, it is important to avoid trauma to the The experienced LPN is capable of gathering
rectal tissue, which could cause bleeding (e.g., data and making observations, including noting
avoid rectal thermometers and enemas). All of breath sounds and performing pulse oximetry.
the other instructions are appropriate to the care Administering medications, such as those
of a patient receiving anticoagulants. delivered via MDIs, is within the scope of
3. Answer: A. Perform endotracheal practice of the LPN. Independently completing
intubation and initiate mechanical ventilation the admission assessment, initiating the nursing
A non-rebreather mask can deliver nearly 100% care plan, and evaluating a patients abilities
oxygen. When the patients oxygenation status require additional education and skills. These
does not improve adequately in response to actions are within the scope of practice of the
delivery of oxygen at this high concentration, professional RN.
refractory hypoxemia is present. Usually at this 9. Answer: C. A 72-year old who needs
stage, the patient is working very hard to teaching about the use of incentive
breathe and may go into respiratory arrest spirometry
unless health care providers intervene by Many surgical patients are taught about
providing intubation and mechanical ventilation coughing, deep breathing, and use of incentive
to decrease the patients work of breathing. spirometry preoperatively. To care for the patient
4. Answer: A. Assisting the patient to sit up with TB in isolation, the nurse must be fitted for
on the side of the bed a high-efficiency particulate air (HEPA)
Assisting patients with positioning and activities respirator mask. The bronchoscopy patient
of daily living is within the educational needs specialized procedure, and the ventilator-
preparation and scope of practice of a nursing dependent patient needs a nurse who is familiar
assistant. Teaching, instructing, and assessing with ventilator care. Both of these patients need
patients all require additional education and experienced nurses.
skills and are more appropriate for a licensed 10. Answer: B. Manually ventilate the patient
nurse. while assessing possible reasons for the
5. Answer: C. Remind the patient to sleep on high-pressure alarm
his side instead of his back. Manual ventilation of the patient will allow you to
The nursing assistant can remind patients about deliver an FiO2 of 100% to the patient while you
actions that have already been taught by the attempt to determine the cause of the high-
nurse and are part of the patients plan of care. pressure alarm. The patient may need
Discussing and teaching require additional reassurance, suctioning, and/or insertion of an
education and training. These actions are within oral airway, but the first step should be
the scope of practice of the RN. The RN can assessment of the reason for the high-pressure
delegate administration of medication to an alarm and resolution of the hypoxemia.
LPN/LVN. 11. Answer: A. Suggest that the patients
6. Answer: D. A 50-year old with asthma who oxygen be humidified
complains of shortness of breath after using When the oxygen flow rate is higher than 4
a bronchodilator L/min, the mucous membranes can be dried out.
The best treatment is to add humidification to lung stiffness that is causing his respiratory
the oxygen delivery system. Application of a distress. Morphine sulfate will only decrease the
water-soluble jelly to the nares can also help respiratory drive and further contribute to his
decrease mucosal irritation. None of the other hypoxemia.
options will treat the problem. 17. Answer: D. Auscultating the lungs for
12. Answer: A. Observe how well the patient crackles.
performs pursed-lip breathing An LPN who has been trained to auscultate
Experienced LPNs/LVNs can use observation of lungs sounds can gather data by routine
patients to gather data regarding how well assessment and observation, under supervision
patients perform interventions that have already of an RN. Independently evaluating patients,
been taught. Assisting patients with ADLs is assessing for symptoms of respiratory failure,
more appropriately delegated to a nursing and monitoring and interpreting laboratory
assistant. Planning and consulting require values require additional education and skill,
additional education and skills, appropriate to an appropriate to the scope of practice of the RN.
RN. 18. Answer: A. Warfarin (Coumadin) 1.0 mg
13. Answer: D. Oral temperature of 101.2 F by mouth (PO).
(38.4C) Medication safety guidelines indicate that use of
A patient who did not have the pneumonia a trailing zero is not appropriate when writing
vaccination or flu shot is at increased risk for medication orders because the order can easily
developing pneumonia or influenza. An elevated be mistaken for a larger dose, such as 10 mg.
temperature indicates some form of infection, The order should be clarified before
which may be respiratory in origin. All of the administration. The other orders are appropriate,
other vital sign values are slightly elevated but based on the patients diagnosis.
are not a cause for immediate concern. 19. Answer: A, C, B, E, D, F.
14. Answer: B. Surfactant (Exosurf) Before each use, the cap is removed and the
Exosurf neonatal is a form of synthetic inhaler is shaken according to the instructions in
surfactant. An infant with RDS may be given two the package insert. Next the patient should tilt
to four doses during the first 24 to 48 hours after the head back and breathe out completely. As
birth. It improves respiratory status, and the patient begins to breathe in deeply through
research has show a significant decrease in the the mouth, the canister should be pressed down
incidence of pneumothorax when it is to release one puff (dose) of the medication. The
administered. patient should continue to breathe in slowly over
15. Answer: B. I will continue to take my 3 to 5 seconds and then hold the breath for at
isoniazid until I am feeling completely well. least 10 seconds to allow the medication to
Patients taking isoniazid must continue the drug reach deep into the lungs. The patient should
for 6 months. The other 3 statements are wait for at least 1 minute between puffs from the
accurate and indicate understanding of TB. inhaler.
Family members should be tested because of 20. Answers. A and B.
their repeated exposure to the patient. Covering The new RN is at an early point in her
the nose and mouth when sneezing or coughing, orientation. The most appropriate patients to
and placing the tissues in plastic bags help assign to her are those in stable condition who
prevent transmission of the causative organism. require routine care. The patient with the
The dietary changes are recommended for lobectomy will require the care of a more
patients with TB. experienced nurse, who will perform frequent
16. Answer: D. Switch the patient to a assessments and monitoring for postoperative
nonrebreather mask at 95% to 100% oxygen complications. The patient admitted with newly
and call the physician to discuss the diagnosed esophageal cancer will also benefit
patients status. from care by an experienced nurse. This patient
The patients history and symptoms suggest the may have questions and needs a
development of ARDS, which will require comprehensive admission assessment. As the
intubation and mechanical ventilation. The new nurse advances through her orientation,
maximum oxygen delivery with a nasal cannula you will want to work with her in providing care
is an Fio2 of 44%. This is achieved with the for these patients with more complex needs.
oxygen flow at 6 L/min, so increasing the flow to
10 L/min will not be helpful. Helping the patient Prioritization, Delegation and Assignment 2
to cough and deep breathe will not improve the Answers and Rationale
1. Answer: D. A client who had a total hip A client with airway problems should be
replacement two days ago and needs blood attended first.
glucose monitoring. 13. Answer: A. Determine the level of
A nurse from the medical-surgical floor floated to consciousness.
the orthopedic unit should be given clients with Assessing the level of consciousness should be
stable condition as those have care similar to the first action when dealing with clients that
her training and experience. A client who is in might have fell over.
postoperative state is more likely to be on a 14. Answer: D. Contact the nursing
stable condition. supervisor to address the situation.
2. Answer: B. Decontaminate the open The nurse should use proper channel of
wound on the clients thigh. communication. The nursing supervisor is
Decontaminating an open wound is the first responsible for the actions of the different
priority for the client. This minimizes absorption members of the nursing team.
of radiation in the clients body. 15. Answer: D. Pneumococcal vaccine.
3. Answer: A. respiratory status. Pneumococcal vaccine is a priority immunization
Assessing respiratory status is the first priority. amongst elderly especially those with chronic
Remember ABC. illnesses. It is administered every five (5) years.
4. Answer: B. Broccoli.
Broccoli are gas forming and therefore, should Prioritization, Delegation and Assignment 3
be avoided. Answers and Rationale
5. Answer: B. Assist with silver nitrate 1. Answer: A. The patient was recently in a
application to the cervix to control bleeding. motor vehicle accident
The priority nursing action when caring for a Rationale: Patients who have recently
client who underwent colposcopy is to assist in experienced trauma are at risk for deep vein
controlling potential bleeding by applying silver thrombosis and pulmonary embolus. None of the
nitrate to the cervix. other findings are risk factors for pulmonary
6. Answer: C. A 35-year-old male with embolus. Prolonged immobilization is also a risk
tracheostomy and copious secretions. factor for DVT and pulmonary embolus, but this
The patient with problem of the airway should be period of bed rest was very short.
given highest priority. Remember Airway, 2. Answer: B. Taking vital signs and pulse
Breathing, and Circulation (ABC) is a priority. oximetry readings every 4 hours
7. Answer: B. Establish whether the client is Rationale: The nursing assistants educational
responsive. preparation includes measurement of vital signs,
Assess first for responsiveness. and an experienced nursing assistant would
8. Answer: D. Lack of trust to the members of know how to check oxygen saturation by pulse
the healthcare team. oximetry. Assessing and observing the patient,
Lack of trust is the common reason for as well as checking ventilator settings, require
reluctance in delegation of tasks. the additional education and skills of the RN
9. Answer: B. wound infection. 3. Answer: C. Maintain the head of the bed at
Wound infection is the most common a 30 to 45-degree angle
complication among obese clients who had Rationale: Research indicates that nursing
undergone surgery. This is due to their poor actions such as maintaining the head of the bed
blood supply in their adipose tissues. at 30 to 45 degrees decrease the incidence of
10. Answer: D. Administer oxygen via nasal VAP. These actions are part of the standard of
cannula. care for patients who require mechanical
Promotion of adequate oxygenation is the most ventilation. The other actions are also
vital to life and therefore should be given highest appropriate for this patient but will not decrease
priority by the nurse. the incidence of VAP
11. Answer: D. Speak with the parents about 4. Answer: B. The patients respiratory rate is
how the fracture occurred. 8 breaths/min.
In case of injury, especially among children, it is Rationale: For patients with chronic
very important that the nurse should first assess emphysema, the stimulus to breathe is a low
possible abuse. Abuse is one of the reporting serum oxygen level (the normal stimulus is a
responsibilities of the nurse. high carbon dioxide level). This patients oxygen
12. Answer: C. A client with chronic flow is too high and is causing a high serum
bronchitis on nasal oxygen. oxygen level, which results in a decreased
respiratory arrest. Crackles, barrel chest, and remind and encourage the patient to take in
assumption of a sitting position leaning over the adequate nutrition. Instructing patients and
night table are common in patients with chronic planning activities require more education and
emphysema skill, and are appropriate to the RNs scope of
5. Answer: C. Reminding the patient to use practice. Monitoring the patients cardiovascular
an incentive spirometer every 1 to 2 hours response to activity is a complex process
while awake requiring additional education, training, and skill,
Rationale: A nursing assistant can remind the and falls within the RNs scope of practice
patient to perform actions that are already part 10. Answer: B. Continuous bubbling in the
of the plan of care. Assisting the patient into the water seal chamber
best position to facilitate coughing requires Rationale: Continuous bubbling indicates an air
specialized knowledge and understanding that is leak that must be identified. With the physicians
beyond the scope of practice of the basic order you can apply a padded clamp to the
nursing assistant. However, an experienced drainage tubing close to the occlusive dressing.
nursing assistant could assist the patient with If the bubbling stops, the air leak may be at the
positioning after the nursing assistant and the chest tube insertion, which will require you to
patient had been taught the proper technique. notify the physician. If the air bubbling does not
The nursing assistant would still be under the stop when you apply the padded clamp, the air
supervision of the RN. Teaching patients about leak is between the clamp and the drainage
adequate fluid intake and techniques that system, and you must assess the system
facilitate coughing requires additional education carefully to locate the leak. Chest tube drainage
and skill, and is within the scope of practice of of 10 to 15 mL/hr is acceptable. Chest tube
the RN dressings are not changed daily but may be
6. Answer: C. The patient is unable to reinforced. The patients complaints of pain need
remember her husbands first name to be assessed and treated. This is important
Rationale: Confusion in a patient this age is but is not as urgent as investigating a chest tube
unusual and may be an indication of leak.
intracerebral bleeding associated with 11. Answer: C. Removing the inner cannula
enoxaparin use. The right leg symptoms are and cleaning using universal precautions
consistent with a resolving deep vein Rationale: When tracheostomy care is
thrombosis; the patient may need teaching performed, a sterile field is set up and sterile
about keeping the right leg elevated above the technique is used. Standard precautions such as
heart to reduce swelling and pain. The presence washing hands must also be maintained but are
of ecchymoses may point to a need to do more not enough when performing tracheostomy care.
patient teaching about avoiding injury while The presence of a tracheostomy tube provides
taking anticoagulants but does not indicate that direct access to the lungs for organisms, so
the physician needs to be called. sterile technique is used to prevent infection. All
7. Answer: A, B, D, and E. of the other steps are correct and appropriate.
Rationale: Bedding should be washed in hot 12. Answers: B, C, D, and E.
water to destroy dust mites. All of the other Rationale: The correct position for a patient with
points are accurate and appropriate to a an anterior nosebleed is upright and leaning
teaching plan for a patient with a new diagnosis forward to prevent blood from entering the
of asthma. stomach and avoid aspiration. All of the other
8. Answer: C. Pneumothorax instructions are appropriate according to best
Rationale: The most common complications practice for emergency care of a patient with an
after birth for infants with RDS is pneumothorax. anterior nosebleed
Alveoli rupture and air leaks into the chest and 13. Answer: C. Marking the tube 1 cm from
compresses the lungs, which makes breathing where it touches the incisor tooth or nares
difficult Rationale: The endotracheal tube should be
9. Answer: B. Encouraging, monitoring, and marked at the level where it touches the incisor
recording nutritional intake tooth or nares. This mark is used to verify that
Rationale: The nursing assistants training the tube has not shifted. The other three actions
includes how to monitor and record intake and are appropriate after endotracheal placement.
output. After the nurse has taught the patient The priority at this time is to verify that the tube
about the importance of adequate nutritional has been correctly placed.
intake for energy, the nursing assistant can
14. Answer: C. The patients blood pressure Baby Angela has metabolic acidosis due to
is 100/48 mm Hg and her heart rate is 102 decreased HCO3 and slightly acidic pH. Her pH
beats/ min value is within the normal range which made the
Rationale: Removal of large quantities of fluid result fully compensated.
from the pleural space can cause fluid to shift 7. Answer: B. Metabolic Alkalosis,
from the circulation into the pleural space, Uncompensated
causing hypotension and tachycardia. The The postoperative clients ABG results show that
patient may need to receive IV fluids to correct he has metabolic alkalosis because of an
this. The other data indicate that the patient increased pH and HCO3. It is uncompensated
needs ongoing monitoring and/or interventions due to the normal PaCO2 which is within 35 to
but would not be unusual findings for a patient 45 mmHg.
with this diagnosis or after this procedure 8. Answer: C. Respiratory Alkalosis,
15. Answer: A. Frequent swallowing Uncompensated
Rationale: Frequent swallowing after a The results show that client Z has respiratory
tonsillectomy may indicate bleeding. You should alkalosis since there is an increase in the pH
inspect the back of the throat for evidence of value and a decrease in PaCO2 which are both
bleeding. The other assessment results are not basic. It is uncompensated due to the normal
unusual in a 3-year old after surgery HCO3 which is within 22-26 mEq/L.
9. Answer: B. Respiratory Acidosis, Fully
Acid Base Balance Analysis 1 Compensated
Answers and Rationale The patient has respiratory acidosis (raised
1. Answer: B. Respiratory Acidosis, Partially carbon dioxide) resulting from asthma and
Compensated respiratory distress syndrome, with
The patient has respiratory acidosis (raised compensation having normal pH value within
carbon dioxide) resulting from an acute 7.35to 7.45, increased PaCO2 which is acidic
exacerbation of chronic obstructive pulmonary and increased HCO3 which is basic.
disease, with partial compensation. 10. Answer: C. Respiratory Acidosis
2. Answer: D. Metabolic Acidosis, Partially, One of the risk factors of having respiratory
Compensated acidosis is hypoventilation which may be due to
The student was diagnosed having diabetes brain trauma, coma, and hypothyroidism or
mellitus. The results show that he has metabolic myxedema. Other risk factors include COPD,
acidosis (low HCO3 -) with respiratory Respiratory conditions such as pneumothorax,
compensation (low CO2). pneumonia and status asthmaticus. Drugs such
3. Answer: A. Respiratory Alkalosis, as Morphine and MgSO4 toxicity are also risk
Uncompensated factors of respiratory acidosis.
The primary disorder is acute respiratory
alkalosis (low CO2) due to the pain and anxiety Acid Base Balance Analysis 2
causing her to hyperventilate. There has not Answers and Rationale
been time for metabolic compensation. 1. Answer: D. Metabolic Alkalosis
4. Answer: C. Metabolic Alkalosis, Vomiting, hypokalemia, overdosage of NaHCO3
Uncompensated and NGT suctioning are considered risk factors
The primary disorder is uncompensated of metabolic alkalosis.
metabolic alkalosis (high HCO3 -). As CO2 is 2. Answer: C. 7.40
the strongest driver of respiration, it generally Normal blood pH must be maintained within a
will not allow hypoventilation as compensation narrow range of 7.35-7.45 to ensure the proper
for metabolic alkalosis. functioning of metabolic processes and the
5. Answer: B. Respiratory Acidosis, delivery of the right amount of oxygen to tissues.
Uncompensated Acidosis refers to an excess of acid in the blood
The results show that Mrs. Johansson has that causes the pH to fall below 7.35, and
respiratory acidosis because of decreased pH alkalosis refers to an excess of base in the blood
and increased PaCO2 which mean acidic in that causes the pH to rise above 7.45.
nature. Meanwhile, it is uncompensated 3. Answer. C. Metabolic Acidosis
because HCO3 is within the normal range. Salicylate overdose causes a high anion gap
6. Answer: C. Metabolic Acidosis, Fully metabolic acidosis in both children and adults.
Compensated Adults commonly develop a mixed acid-base
disorder as a respiratory alkalosis due to direct
respiratory centre stimulation occurs as well. carbon dioxide levels are so high that they no
This second disorder is uncommon in children. longer stimulate respirations but depress them.
4. Answer: B. 36 mm Hg 3. Answer: A. Metabolic Acidosis
The normal range for PaCO2 is from 35 to 45 The body compensates by using body fat for
mm Hg. energy, producing abnormal amounts of ketone
5. Answer: B. Respiratory Alkalosis bodies. In an effort to neutralize the ketones and
Hyperventilation is typically the underlying cause maintain the acid-base balance of the body,
of respiratory alkalosis. Hyperventilation is also plasma bicarbonate is exhausted. This condition
known as overbreathing. When someone is can develop in anyone who does not eat an
hyperventilating, they tend to breathe very adequate diet and whose body fat must be
deeply or very rapidly. burned for energy. Symptoms include headache
6. Answer: B. 24 mmol/L and mental dullness.
The normal value for bicarbonate (HCO3) is 22- 4. Answer: C. Metabolic Alkalosis
26 mmol/L or mEq/L. It may vary slightly among In metabolic alkalosis, breathing becomes
different laboratories. The given values show the depressed in an effort to conserve carbon
common measurement range of results for these dioxide for combination with water in the blood
tests. Some laboratories use different to raise the blood level of carbonic acid.
measurements or may test different specimens. Symptoms include confusion, dizziness,
7. Answer: D. Metabolic Alkalosis numbness or tingling of fingers or toes.
NGT suctioning, vomiting, hypokalemia and 5. Answer: A. Kidneys and Lungs
overdosage of NaHCO3 are considered risk The carbonic acid concentration is controlled by
factors of metabolic alkalosis. the amount of carbon dioxide excreted by the
8. Answer: B. The pH will rise and PCO2 will lungs. The bicarbonate concentration is
fall. controlled by the kidneys, which selectively
The mountaineer will suffer from a respiratory retain or excrete bicarbonate in response to the
alkalosis. The decline in the PO2 with altitude bodys needs.
will stimulate breathing to offset the hypoxia. 6. Answer: A. True
Carbon dioxide is driven from the blood faster ABGs are blood tests that are useful in
than it is produced in the tissues so PCO2 falls identifying the cause and extent of the acid-base
and pH rises. disturbance and in guiding and monitoring
9. Answer: C. Simultaneous Respiratory and treatment.
Metabolic Acidosis 7. Answer: B. False
Whenever the PCO2 and HCO3 are abnormal in The major effect is a depression of the central
opposite directions, ie, one above normal while nervous system, as evidenced by disorientation
the other is reduced, a mixed respiratory and followed by coma.
metabolic acid-base disorder exists. When the 8. Answer: A. True
PCO2 is elevated and the [HCO3-] reduced, The muscles may go into a state of tetany and
respiratory acidosis and metabolic acidosis convulsions.
coexist. 9. Answer: A. True
10. Answer: A. Hyperthermia The maintenance of acid-base balance, which in
An individual is considered to have hyperthermia one part of homeostasis, is evidenced by an
if he or she has a temperature of >37.5 or 38.3 arterial plasma pH value of 7.35-7.45. Many
C (99.5 or 100.9 F). Measurement of arterial mechanisms in the body work together to
blood gases are normal. achieve and maintain this delicate narrow range
of pH that is essential for normal cell function.
Acid Base Balance Analysis 3 10. Answer: B. False
Answers and Rationale Acids are substances having one or more
1. Answer: B. Respiratory Alkalosis hydrogen ions that can be liberated into a
Excessive pulmonary ventilation decreases solution.
hydrogen ion concentration and thus causes Bases are substances that can bind hydrogen
respiratory alkalosis. It can become dangerous ions in a solution.
when it leads to cardiac dysrhythmias caused 11. Answer: D. Respiratory Alkalosis,
partly by a decrease in serum potassium levels. Partially Compensated
2. Answer: D. Respiratory Acidosis 12. Answer: D. Normal
An excess of carbon dioxide (hypercapnia) can 13. Answer: B. Respiratory Alkalosis,
cause carbon dioxide narcosis. In this condition, Uncompensated
14. Answer: D. Respiratory Acidosis, 8. Answer: D. Leadership is shared at the
Partially Compensated point of care.
15. Answer: B. Metabolic Acidosis, Partially Shared governance allows the staff nurses to
Compensated have the authority, responsibility and
16. Answer: B. Respiratory Alkalosis, accountability for their own practice.
Partially Compensated 9. Answer: D. Inspires others with vision
17. Answer: A. Metabolic Alkalosis, Fully Inspires others with a vision is characteristic of a
Compensated transformational leader. He is focused more on
18. Answer: A. Metabolic Alkalosis, Partially the day-to-day operations of the
Compensated department/unit.
19. Answer: D. Respiratory Alkalosis, 10. Answer: A. Have condescending trust
Uncompensated and confidence in their subordinates
20. Answer: C. Respiratory Acidosis, Fully Benevolent-authoritative managers pretentiously
Compensated show their trust and confidence to their followers
11. Answer: A. Call for a staff meeting and
Nursing Leadership and Management 1 take this up in the agenda.
This will allow for the participation of every staff
1. Answer: C. Country Club Management in the unit. If they contribute to the solutions of
Country club management style puts concern for the problem, they will own the solutions; hence
the staff as number one priority at the expense the chance for compliance would be greater.
of the delivery of services. He/she runs the 12. Answer: C. Low morale of staff in
department just like a country club where his unit
everyone is happy including the manager. Low morale of staff is an internal factor that
2. Answer: C. Servant leader affects only the unit. All the rest of the options
Servant leaders are open-minded, listen deeply, emanate from the top executive or from outside
try to fully understand others and not being the institution.
judgmental 13. Answer: B. Majority rule
3. Answer: A. Possesses inspirational quality Majority rule involves dividing the house and the
that makes followers gets attracted of him highest vote wins. 1/2 + 1 is a majority.
and regards him with reverence 14. Answer: B. system used to deliver care
Charismatic leaders make the followers feel at A system used to deliver care. In the 70s it was
ease in their presence. They feel that they are in termed as methods of patient assignment; in the
good hands whenever the leader is around. early 80s it was called modalities of patient care
4. Answer: C. Assessment of personal traits then patterns of nursing care in the 90s until
is a reliable tool for predicting a managers recently authors called it nursing care systems.
potential. 15. Answer: A. Concentrates on tasks and
It is not conclusive that certain qualities of a activities
person would make him become a good Functional nursing is focused on tasks and
manager. It can only predict a managers activities and not on the holistic care of the
potential of becoming a good one. patients
5. Answer: A. Recognizes staff for going 16. Answer: A. Psychological and
beyond expectations by giving them sociological needs are emphasized.
citations When the functional method is used, the
Path Goal theory according to House and psychological and sociological needs of the
associates rewards good performance so that patients are neglected; the patients are regarded
others would do the same as tasks to be done
6. Answer: C. Great Man 17. Answer: A. Assessing nursing needs and
Leaders become leaders because of their birth problems
right. This is also called Genetic theory or the This option follows the framework of the nursing
Aristotelian theory process at the same time applies the
7. Answer: C. Laissez faire management process of planning, organizing,
Laissez faire leadership is preferred when the directing and controlling
followers know what to do and are experts in the 18. Answer: B. Preparing a nursing care plan
field. This leadership style is relationship- in collaboration with the patient
oriented rather than task-centered. The best source of information about the priority
needs of the patient is the patient himself.
Hence using a nursing care plan based on his 29. Answer: A. Increase the patient
expressed priority needs would ensure meeting satisfaction rate
his needs effectively. Goal is a desired result towards which efforts
19. Answer: D. Patients who need the most are directed.
care 30. Answer: D. Organizational culture
In setting priorities for a group of patients, those An organizational culture refers to the way the
who need the most care should be number-one members of the organization think together and
priority to ensure that their critical needs are met do things around them together. Its their way of
adequately. The needs of other patients who life in that organization
need less care ca be attended to later or even 31. Answer: A. Proactive and caring with one
delegated to assistive personnel according to another
rules on delegation. 32. Answer: A. Organizational structure
20. Answer: C. Integrate the solutions to his Organizational structure provides information on
day-to-day activities the channel of authority, (i.e., who reports to
Integrate the solutions to his day-to-day whom and with what authority) the number of
activities is expected to happen during the third people who directly reports to the various levels
stage of change when the change agent of hierarchy and the lines of communication
incorporate the selected solutions to his system whether line or staff.
and begins to create a change. 33. Answer: B. Informal
21. Answer: C. Focuses on routine tasks This is usually not published and oftentimes
Strategic planning involves options A, B and D concealed.
except C which is attributed to operational 34. Answer: D. Tall organization
planning Tall organizations are highly centralized
22. Answer: A. The Holy Spirit Medical organizations where decision making is centered
Center is a trendsetter in tertiary health care on one authority level.
in the next five years 35. Answer: A. 1 & 2
A vision refers to what the institution wants to Centralized organizations are needs only a few
become within a particular period of time. managers hence they are less expensive and
23. Answer: B. Goal easier to manage
24. Answer: C. Broken line 36. Answer: C. having legitimate right to act
This is a staff relationship hence it is depicted by Authority is a legitimate or official right to give
a broken line in the organizational structure command. This is an officially sanctioned
25. Answer: C. Unity of command responsibility
The principle of unity of command means that 37. Answer: B. Provide a pair of hands to
employees should receive orders coming from other units as needed
only one manager and not from two managers. Providing a pair of hands for other units is not a
This averts the possibility of sowing confusion purpose in doing an effective staffing process.
among the members of the organization This is a function of a staffing coordinator at a
26. Answer: B. Hierarchy centralized model.
Hierarchy refers to the pattern of reporting or the 38. Answer: D. Staff preferences
formal line of authority in an organizational Staff preferences should be the least priority in
structure. formulating objectives of nursing care. Individual
27. Answer: B. Unity of direction preferences should be subordinate to the
Unity of direction means having one goal or one interest of the patients.
objective for the team to pursue; hence all 39. Answer: A. Uses visioning as the
members of the organization should put their essence of leadership.
efforts together towards the attainment of their Transformational leadership relies heavily on
common goal or objective. visioning as the core of leadership.
28. Answer: A. Lets work together in 40. Answer: C. Team management
harmony; we need to be supportive of one Team management has a high concern for
another services and high concern for staff.
The principle of esprit d corps refers to
promoting harmony in the workplace, which is Nursing leadership and management 2
essential in maintaining a climate conducive to 1. Answer: A. Identify the source of the
work. conflict and understand the points of friction
This involves a problem solving approach, which Staffing is a management function involving
addresses the root cause of the problem. putting the best people to accomplish tasks and
2. Answer: B. Is not beneficial; hence it activities to attain the goals of the organization.
should be prevented at all times 12. Answer: B. Record of related learning
Conflicts are beneficial because it surfaces out experience (RLE)
issues in the open and can be solved right away. Record of RLE is not required for employment
Likewise, members of the team become more purposes but it is required for the nurses
conscientious with their work when they are licensure examination.
aware that other members of the team are 13. Answer: B. Induction
watching them. This step in the recruitment process gives time
3. Answer: C. Avoidance for the staff to submit all the documentary
This strategy shuns discussing the issue head- requirements for employment.
on and prefers to postpone it to a later time. In 14. Answer: B. Decentralized
effect the problem remains unsolved and both Decentralized structures allow the staff to make
parties are in a lose-lose situation. decisions on matters pertaining to their practice
4. Answer: D. Let the staff ventilate her and communicate in downward, upward, lateral
feelings and ask how she can be of help. and diagonal flow.
Reaching out and helping the staff is the most 15. Answer: C. Rightmost box
effective strategy in dealing with burn out. The leftmost box is occupied by the highest
Knowing that someone is ready to help makes authority while the lowest level worker occupies
the staff feel important; hence her self-worth is the rightmost box.
enhanced. 16. Answer: D. Promotes better interpersonal
5. Answer: D. Focusing activity on the relationship
correction of identified behavior. Decentralized structures allow the staff to solve
Performance appraisal deal with both positive decisions by themselves, involve them in
and negative performance; is not meant to be a decision making; hence they are always given
fault-finding activity opportunities to interact with one another.
6. Answer: C. Patients are the best source of 17. Answer: B. Provides care to a group of
information regarding personnel appraisal. patients together with a group of nurses
The patient can be a source of information about This function is done in team nursing where the
the performance of the staff but it is never the nurse is a member of a team that provides care
best source. Directly observing the staff is the for a group of patients.
best source of information for personnel 18. Answer: B. Modular nursing
appraisal. Modular nursing is a variant of team nursing.
7. Answer: C. The evaluation is focused on The difference lies in the fact that the members
objective data systematically. in modular nursing are paraprofessional
Collecting objective data systematically can not workers.
be achieved in an informal appraisal. It is 19. Answer: B. Identify the values of the
focused on what actually happens in the natural department
work setting. Identify the values of the department will set the
8. Answer: D. The session is private between guiding principles within which the department
the two members. will operate its activities
The session is private between the manager and 20. Answer: B. Rotation of duty will be done
the staff and remains to be so when the two every four weeks for all patient care
parties do not divulge the information to others. personnel.
9. Answer: B. Consultative Structure standards include management
A consultative manager is almost like a system, facilities, equipment, materials needed
participative manager. The participative to deliver care to patients. Rotation of duty is a
manager has complete trust and confidence in management system.
the subordinate, always uses the opinions and 21. Answer: A. Process
ideas of subordinates and communicates in all Process standards include care plans, nursing
directions. procedure to be done to address the needs of
10. Answer: D. Unity of direction the patients.
Unity of direction is a management principle, not 22. Answer: D. Identify possible courses of
an element of an organizational structure. action
11. Answer: A. Staffing
This is a step in a quality control process and Reliability is repeatability of the instrument; it
not a basic step in the control process. can elicit the same responses even with varied
23. Answer: B. Characteristics used to administration of the instrument
measure the level of nursing care 36. Answer: A. Sensitivity
Criteria are specific characteristics used to Sensitivity is an attribute of the instrument that
measure the standard of care. allow the respondents to distinguish differences
24. Answer: A. Instructing the members of of the options where to choose from
the standards committee to prepare policies 37. Answer: A. Validity
Instructing the members involves a directing Validity is ensuring that the instrument contains
function. appropriate questions about the research topic
25. Answer: C. Patients reports 95% 38. Answer: A. Random
satisfaction rate prior to discharge from the Random sampling gives equal chance for all the
hospital. elements in the population to be picked as part
This refers to an outcome standard, which is a of the sample.
result of the care that is rendered to the patient. 39. Answer: B. Ethnography
26. Answer: A. The things that were planned Ethnography is focused on patterns of behavior
are done of selected people within a culture
Controlling is defined as seeing to it that what is 40. Answer: B. Madeleine Leininger
planned is done. Madeleine Leininger developed the theory on
27. Answer: A. Span of control transcultural theory based on her observations
Span of control refers to the number of workers on the behavior of selected people within a
who report directly to a manager. culture.
28. Answer: B. Initiate a group interaction
Initiate a group interaction will be an opportunity
to discuss the problem in the open.
29. Answer: C. Age of patients
An extraneous variable is not the primary
concern of the researcher but has an effect on
the results of the study. Adult patients may be
young, middle or late adult.
30. Answer: B. Sr. Callista Roy
Sr. Callista Roy developed the Adaptation Model
which involves the physiologic mode, self-
concept mode, role function mode and
dependence mode
31. Answer: C. Most accurate and valid
method of data gathering
The most serious disadvantage of this method is
accuracy and validity of information gathered
32. Answer: B. Turnaround Time in
Emergency Rooms
The article is for pediatric patients and may not
be relevant for adult patients.
33. Answer: C. Salary of nurses
Salary of staff nurses is not an indicator of
patient satisfaction, hence need not be included
as a variable in the study.
34. Answer: A. degree of agreement and
disagreement
Likert scale is a 5-point summated scale used to
determine the degree of agreement or
disagreement of the respondents to a statement
in a study.
35. Answer: B. Reliability

Você também pode gostar